Sei sulla pagina 1di 99

Done by Dr.

arafat alwadan my greating for u all

AAOS
Adult spine self
Assessment 2018

Gunphil &Thiotacid
Enclex
‫ﺍﻟﺸﺮﻛﻪ ﺍﻟﻌﺮﺑﻴﻪ‬
‫ﺷﺮﻛﺔ ﺍﻟﻤﻴﺪﺍﻥ‬

1
Done by Dr.arafat alwadan my greating for u all

Question 1 of 100
When compared with posterior decompression and fusion, the addition of
an interbody fusion for the treatment of degenerative spondylolisthesis and
stenosis has been shown to
A. result in increased patient functional outcome scores.
B. reduce the incidence of symptomatic pseudarthrosis.
C. increase the length of hospital stay.
D. increase hospital costs.
Discussions: D

The use of an interbody graft has been shown to increase hospital costs.
Gottschalk and associates found no change in Oswestry Disability Index (ODI)
or 36-Item Short-Form Health Survey (SF-36) scores when comparing
patients fused using either posterior fusion or transforaminal interbody
fusion. They also found no change in fusion rates at 3 years after surgery.
Carreon and associates showed some that using a posterior place interbody
transforaminal lumbar interbody fusion (TLIF) or posterior lumbar interbody
fusion (PLIF) did result in improved ODI and SF-6D scores but did not result
in any change in EuroQol five dimensions questionnaire (EQ-5D) scores.
Using the EQ-5D data, they estimated that the use of an interbody graft
becomes cost prohibitive if the charges exceed $1,570 above the cost of a
posterior fusion. The use of an interbody cage has not been shown to
increase hospital stay.
Question 2 of 100

Improvements in outcomes after correction surgery for adult spinal


deformity (ASD) are lowered by
A. baseline depression.
B. race.
C. age.
D. ethnicity.
Discussions: A

2
Done by Dr.arafat alwadan my greating for u all

Depression increases the risk of poor outcomes in spine surgery, including


ASD surgery. Race, age, and ethnicity have not been shown to have
substantial effects.
Question 3 of 100

What structure is most at risk when dissecting too far laterally during a C1-2
posterior fusion?

A. Spinal cord
B. Transverse ligament
C. Sympathetic chain
D. Vertebral artery

Discussion: D

Surgical dissection of the posterior arch of C1 should be limited to 1.5 cm


lateral to the midline to avoid injury to the vertebral artery. Injury to the spinal
cord would be a risk of dissecting too far ventrally past the posterior arch of
C1. The transverse ligament is located ventral to the spinal cord and would
not be at risk during a posterior exposure. The sympathetic chain is an
anterior structure and is at risk during anterior approaches.
Question 4 of 100

A 14-year-old, skeletally immature girl has adolescent idiopathic scoliosis


(AIS). Her major curve resides in the thoracic spine, and it measures 28˚. She
has been advised to wear a brace but is not willing to do so and inquires
about the relative success rate of bracing versus close observation. The
patient should be told that, when compared with observation, bracing
A. does not reduce the progression of AIS curves.
B. reduces the need for surgery by 50%.
C. reduces the need for surgery by 75%.
D. reduces the need for surgery by 25%

Discussion: B

3
Done by Dr.arafat alwadan my greating for u all

The odds of needing surgery are reduced by 2 to 1 compared with


observation alone, according to a recent prospective randomized study by
Weinstein and associates. The study was ended early when the efficacy of
bracing reached a predetermined end point. Bracing was more effective
when the brace was worn at least 12 hours per day.
Question 5 of 100

Figures 1 and 2 are CT scans obtained from a 68-year-old man who has had
progressive neck pain and stiffness, worsening gait imbalance, upper
extremity weakness, early muscle fatigue, difficulty with fine motor control,
and difficulty with activities of daily living over the past few years. On physical
examination, he has a wide based stiff legged gait, generalized upper
extremity weakness, dense sensory loss in the upper and lower extremities,
and markedly brisk reflexes. What is the most appropriate treatment for this
patient?

A. Observation
B. Cervical epidural injections
C. Multilevel anterior cervical decompression and fusion
D. Posterior cervical laminoplasties from C3-6
Discussions: D

This patient has progressive myelopathy secondary to ossification of the


posterior longitudinal ligament. Diagnostic imaging reveals multilevel
cervical cord compression from C4-6. The patient has maintained reasonable
cervical lordosis. A posterior procedure such as multilevel laminoplasty

4
Done by Dr.arafat alwadan my greating for u all

decompresses the spine, is motion preserving, and has a low complication


rate. Observation and cervical epidural injections are not viable options in
patients with progressive myelopathy. Anterior cervical decompression,
including corpectomy, is an option; however, anterior procedures have an
increased risk of complications such as dural tear or cerebrospinal fluid leak.
The axial CT image shows a "double layer" sign, which is consistent with dural
ossification and increases the risk of dural injury with anterior decompression
Question 6 of 100

CLINICAL SITUATION

Figure 1 shows a CT from the cervical spine of an 85-year-old woman who


fell from a standing height 1 week earlier. She is independent and ambulatory
and resides in an assisted living facility. She reports persistent neck pain but
denies arm pain or weakness. She is neurologically intact.

Fractures in this region of C2 have a high risk of

A. spinal cord injury.


B. union.
C. nonunion.
D. stroke
Discussion: C

Increased displacement and angulation of odontoid fractures have an


increased risk of nonunion. Surgical stabilization has reduced the risks of
mortality without increasing the risk of complications compared with
nonsurgical treatment options. When treating odontoid fractures, halo vest

5
Done by Dr.arafat alwadan my greating for u all

immobilization has been shown to increase the risk of dysphagia in the


elderly.
Question 7 of 100

CLINICAL SITUATION

Figure 1 shows a CT from the cervical spine of an 85-year-old woman who


fell from a standing height 1 week earlier. She is independent and ambulatory
and resides in an assisted living facility. She reports persistent neck pain but
denies arm pain or weakness. She is neurologically intact.

The preferred mode of treatment for this patient’s fracture is

A. a halo vest.
B. a hard cervical collar.
C. physical therapy.
D. posterior C1-2 arthrodesis.
Discussion: D

Increased displacement and angulation of odontoid fractures have an


increased risk of nonunion. Surgical stabilization has reduced the risks of
mortality without increasing the risk of complications compared with
nonsurgical treatment options. When treating odontoid fractures, halo vest
immobilization has been shown to increase the risk of dysphagia in the
elderly.

6
Done by Dr.arafat alwadan my greating for u all

Question 8 of 100

CLINICAL SITUATION

Figure 1 shows a CT from the cervical spine of an 85-year-old woman who


fell from a standing height 1 week earlier. She is independent and ambulatory
and resides in an assisted living facility. She reports persistent neck pain but
denies arm pain or weakness. She is neurologically intact.

What are the risks associated with halo vest treatment, compared with hard-
collar treatment?

A. Increased mobility
B. Dysphagia
C. Spinal cord injury
D. Dysphonia
Discussion: B

Increased displacement and angulation of odontoid fractures have an


increased risk of nonunion. Surgical stabilization has reduced the risks of
mortality without increasing the risk of complications compared with
nonsurgical treatment
Question 9 of 100

Figure 1 shows a radiograph obtained from a 68-year-old man who was


treated with anterior neck surgery 12 months earlier. Despite some clinical
improvement, he continues to have unsteadiness of gait and generalized

7
Done by Dr.arafat alwadan my greating for u all

clumsiness. He has persistent biceps and triceps weakness. New CT and MRI
scans are obtained, which show residual multilevel cervical stenosis. An
elective posterior cervical procedure is performed. He immediately reports
increased biceps and triceps strength. At his 2-week post operative
appointment, the patient is distraught because of new-onset right dominant
shoulder pain and weakness. He states he cannot brush his teeth or comb
his hair. On physical examination, he is shown to have profound weakness
(grade 2 of 5), with left shoulder abduction and moderate weakness with left
elbow flexion. Sensation is also decreased in the left deltoid region. His
wound is clean and nonindurated and shows no erythema or swelling. He is
afebrile and has a normal white blood count. What is the best next step?

A. Reassurance and observation


B. Emergent CT of the cervical spine
C. EMG and nerve conduction velocitytest
D. Immediate re-exploration of posterior cervical wound
Discussion: A

This patient has a classic C5 nerve root palsy, which can occur up to 1 year
after decompressive anterior or posterior cervical surgery. It has been
reported that intraoperative neuromonitoring and prophylactic C4-5
neuroforaminotomy can reduce the incidence of C5 palsy in posterior
cervical decompressive surgery. The weakness can be debilitating, but
approximately 70% of cases resolve within 6 months. Recovery is
spontaneous, and no treatment has been proven to improve recovery. CT
may be reasonable, and wound re-exploration would be needed if
hematoma, infection, or aberrant screw placement was suspected. In this
scenario, none of these factors seem to be present. Also, the delayed nature

8
Done by Dr.arafat alwadan my greating for u all

of the findings makes screw malplacement less likely. Neurodiagnostic


testing in the early postoperative period (earlier than 6 weeks) would be
nondiagnostic.
Question 10 of 100

A 73-year-old woman has back and leg pain. Imaging reveals a lumbar
degenerative scoliosis. Nonsurgical management, consisting of physical
therapy, medications, and injections, has failed. During the surgical planning,
dual-energy x-ray absorptiometryis performed, and her T-score returns as -
2.6. Intraoperative options to help reduce the risk of instrumentation failure
include
A. teriparatide injection.
B. multilevel interbody fusion.
C. augmentation of pedicle screws with polymethylmethacrylate (PMMA).
D. iliac crest bone graft.
Discussion: C

Instrumentation of the osteoporotic spine is becoming more common as the


population ages. Some intraoperative options to reduce pedicle screw failure
rates include augmenting the pedicle screw with PMMA, using a fenestrated
screw designed for injection of the PMMA through the screw, and using
hydroxyapatite coated screws. Teriparatide is a parathyroid hormone
analogue used as a second-line treatment for osteoporosis. Preoperative
administration potentially can increase bone quality. Postoperative
administration of teriparatide has been shown to increase lumbar fusion
rates. In the setting of osteoporosis, multilevel interbody fusion can increase
the risk of implant subsidence. Although iliac crest bone graft is the gold
standard graft used to obtain fusion, it does not have immediate impact on
the rate of implant failure in osteoporosis.
Question 11 of 100

CLINICAL SITUATION

9
Done by Dr.arafat alwadan my greating for u all

Figures 1 and 2 show sagittal and axial MRI from an 83-year-old woman with
an ataxic gait, loss of dexterity, and intermittent loss of bowel and bladder
control. The symptoms have been getting progressively worse over the past
several weeks.

What disease process is most likely the cause of the patient’s symptoms?

A. Systemic lupus erythematosus


B. Rheumatoid arthritis
C. Diabetes mellitus
D. Pigmented villonodular synovitis
Discussion: B

The patient has a pannus at the C1-2 articulation that is compressing the
spinal cord and causing myelopathy symptoms. The development of a
pannus at this location has been associated with rheumatoid arthritis.
Steinberger and associates showed increased morbidity and mortality when
using an anterior approach and the surgery took longer than 4 hours. Chieng
and associates showed better outcomes and lower complications rates using
a posterior approach.
Question 12 of 100

CLINICAL SITUATION

Figures 1 and 2 show sagittal and axial MRI from an 83-year-old woman with
an ataxic gait, loss of dexterity, and intermittent loss of bowel and bladder
control. The symptoms have been getting progressively worse over the past
several weeks.

10
Done by Dr.arafat alwadan my greating for u all

When discussing anterior versus posterior surgical techniques, you counsel


this patient that

A. the risks of a transoral decompression are higher if the surgery takes longer
than 4 hours.
B. posterior surgery is associated with longer hospital stays.
C. anterior surgery is associated with better postoperative outcomes.
D. posterior surgery is associated with a higher rate of complications.

Discussion: A

The patient has a pannus at the C1-2 articulation that is compressing the
spinal cord and causing myelopathy symptoms. The development of a
pannus at this location has been associated with rheumatoid arthritis.
Steinberger and associates showed increased morbidity and mortality when
using an anterior approach and the surgery took longer than 4 hours. Chieng
and associates showed better outcomes and lower complications rates using
a posterior approach.
Question 13 of 100

After performing an anterior cervical diskectomy and fusion at C5-6, the


patient has moderate neck pain, neck stiffness, and discomfort when
swallowing solid foods. Figure 1 shows a clinical photograph of his
appearance. What has most likely occurred?

11
Done by Dr.arafat alwadan my greating for u all

A. Recurrent laryngeal nerve injury


B. Superior laryngeal nerve injury
C. Ansa cervicalis nerve injury
D. Injury to the sympathetic trunk
Discussion: D

This patient has a Horner syndrome, which is manifested by ptosis, miosis,


anhydrosis, occasional reddish conjunctiva, swelling of the lower eyelid, and
decreased tear production. The recurrent laryngeal nerve innervates all
pharyngeal muscles except the cricothyroid. Injury to this nerve is manifested
by hoarseness. The superior laryngeal nerve supplies the cricothyroid.
Cricothyroid weakness is manifested by voice fatigue and difficulty singing
in high pitch. The ansa cervicalis innervates all the strap muscles except the
thyrohyoid muscle, which is innervated by the hypoglossal nerve.
Question 14 of 100

The use of bone morphogenetic protein-2 (BMP-2) is cost effective


compared with autologous iliac crest bone graft when

A. used in routine spinal arthrodesis.


B. productivity and lost wages are included in the analysis.
C. upfront costs are included in the analysis.
D. used in tumor spinal arthrodesis.
Discussion: B

The use of BMP-2 in lumbar spine surgery is cost effective compared with
autologous iliac crest bone graft, because fewer additional treatments are
needed, including a decreased incidence of revision surgery. This cost
effectiveness is evident when evaluating productivity and lost wages. BMP-2
is contraindicated in tumor surgery.

12
Done by Dr.arafat alwadan my greating for u all

Question 15 of 100

The use of vancomycin powder in spinal surgery has been shown to

A. increase the risk of postoperative wound infection.


B. increase the risk of vancomycin-resistant organisms.
C. have no effect on the rate of postoperative wound infection.
D. reduce the risk of postoperative wound infection.

Discussion: D

In a meta-analysis of 9 studies (8 level III, 1 level II) involving 2,574 cases, with
106 infections in the control group and 33 in the vancomycin group, a relative
risk reduction of 68% was observed. The number needed to treat to prevent
1 infection was 36 cases.
The use of adjuvant vancomycin powder was associated with a significant
reduction in the incidence of postoperative infection and in infection-related
medical costs. These findings suggest that using adjuvant vancomycin
powder in high-risk patients undergoing spinal fusion is a cost-saving option
for preventing postoperative infections. It can lead to cost savings of
$438,165 per 100 spinal fusions performed. The use of vancomycin powder
has not been shown to increase the incidence of vancomycin resistance.
Question 16 of 100

A 73-year-old man with diabetes comes to the emergency department with


low back pain and fever. His blood cultures are positive for methicillin-
resistant Staphylococcus aureus (MRSA). He has no neurologic signs or
symptoms of lumbar radiculopathy or cauda equina syndrome. Figure
1 shows his axial T2-weighted MRI, and an arrow marks the junction of the
theca sac and the epidural abscess. What is the most appropriate
recommendation for this patient?

13
Done by Dr.arafat alwadan my greating for u all

A. weeks, followed by oral antibiotics


B. Intravenous vancomycin for 6 weeks, followed by oral antibiotics
C. Lumbar laminectomy with evacuation of the abscess, followed by antibiotics
D. Anterior diskectomy, followed by antibiotics

Discussion: C

Epidural abscess once was considered an absolute indication for surgery.


Nonsurgical management has been gaining ground for select patients,
however. Kim and associates reported the results of a large series of patients
treated for epidural abscess. Many of the patients were treated successfully
without surgery, and nonsurgical management was chosen for many of the
patients who presented without signs or symptoms of neurologic
dysfunction. The authors identified four risk factors that were highly
associated with the failure of nonsurgical management, however, including
age older than 65 years, diabetes, MRSA, and neurologic compromise.
Question 17 of 100

A 63-year-old woman has a lumbar epidural abscess. She is neurologically


intact. Initially, she is treated with empiric intravenous antibiotics. Which
factor suggests that nonsurgical care is likely to be unsuccessful?
A. C-reactive protein (CRP) level of 100
B. Age 63 years
C. Blood cultures positive for coagulase-negative staphylococci

14
Done by Dr.arafat alwadan my greating for u all

D. White blood cell count (WBC) 13.5


Discussion: D

Risk factors for the failure of nonsurgical care for epidural abscess include a
history of intravenous drug abuse, diabetes, age older than 65 years, CRP
higher than 115, WBC higher than 12.5, and cultures positive for
Staphylococcus aureus. A progressive neurologic deficit should be
considered a sign that nonsurgical care has failed and that surgery should be
considered.
Question 18 of 100

Figures 1 and 2 show a sagittal CT scan and a clinical photograph from a 16-
year-old boy who was found unresponsive after a diving accident. He had to
be extricated from the bottom of a swimming pool by friends and was
intubated at the scene by paramedics. In the emergency department, he was
found to have weak triceps and grip strength bilaterally. Lower extremity
strength was trace (grade 1 of 5). Sensation was diminished below the T2
dermatome. He had an absent bulbocavernosus reflex. Immediate open
posterior reduction and stabilization were performed. Postoperative MRI
revealed no iatrogenic disk herniation. Postoperative neurologic function
revealed grade 4 strength in the triceps, wrist flexors, and bilateral grip. His
lower extremity strength was grade 3. The sensory examination showed only
slight diminution in the legs bilaterally. His bulbocavernosus reflex returned
on postoperative day 2. On postoperative day 3, he required reintubation
and was noted to require high positive end-expiratory pressures and a high
ventilatory rate to keep him oxygenated. On postoperative day 4, his
temperature was 38.7° C, blood pressure was 90/48, and pulse was 110 beats
per minute. The urinary output measured 32 ml per hour. The white blood
cell count was 14.8 with a left shift. A chest CT was ordered, and the results
are shown in Figure 3. What best describes his condition?
A. Neurogenic shock
B. Spinal shock

15
Done by Dr.arafat alwadan my greating for u all

C. Hypovolemic shock
D. Septic shock
Discussion: D

Aspiration pneumonia developed, as seen on CT, and presumably occurred


during his near drowning accident. His laboratory values and hemodynamic
status are characteristic for septic shock. His original neurologic injury is an
incomplete spinal cord injury, but after reduction and stabilization, he
showed neurologic improvement, so this injury should not have contributed
to his circulatory problems. The treatment for septic shock is broad spectrum
antibiotics (species specific if the pathogen is identified) and pressors.
Question 19 of 100

CLINICAL SITUATION

A 68-year-old man has a 6-month history of severe left buttock pain


extending down the back of the thigh. He has a past surgical history of fusion
from L3 to S1 performed 5 years earlier and a left total hip replacement
performed 2 years earlier. Recent radiographs of the hip are unremarkable.
On examination, he is noted to sit leaning towards the right side. The area
just below the left posterior superior iliac spine is tender to palpation. He has
a negative straight leg raise test but does have pain with flexion abduction
and external rotation of the left hip.

Which test is most likely to confirm the cause of the patient’s pain?
A. Intra-articular sacroiliac (SI) joint injection
B. L2-3 selective nerve root block radiculopathy
C. Triple-phase bone scan
D. MRI of the pelvis
Discussion: A

The patient has SI joint pain, which typically is felt in the buttock over the
joint and can cause referred pain into the leg. Physical examination findings
include tenderness over the joint (using the Fortin finger test); pain with

16
Done by Dr.arafat alwadan my greating for u all

compression, distraction, and shear forces across the joint; pain with flexion,
abduction, and external rotation; and pain with the Gaenslen test. The
diagnosis is confirmed with an intra-articular injection. Prior lumbar fusion,
especially when involving more than 3 levels, has been associated with an
increased risk of SI joint pain. If nonsurgical care fails, minimally invasive
fusion of the SI joint can be an option that improves pain and function
outcomes better than continued nonsurgical care, according to several
studies. Polly and associates have shown that the amount of pain relief
following surgery does not correlate to the amount of relief following
injections. Pseudarthrosis rates have been reported to be about 5%.
Incidences of injury to the sacral nerve roots following surgery have been
reported.
Question 20 of 100

CLINICAL SITUATION

A 68-year-old man has a 6-month history of severe left buttock pain


extending down the back of the thigh. He has a past surgical history of fusion
from L3 to S1 performed 5 years earlier and a left total hip replacement
performed 2 years earlier. Recent radiographs of the hip are unremarkable.
On examination, he is noted to sit leaning towards the right side. The area
just below the left posterior superior iliac spine is tender to palpation. He has
a negative straight leg raise test but does have pain with flexion abduction
and external rotation of the left hip.

What risk factor may have been a predisposing factor in the development of
this condition?
A. Prior total hip arthroplasty
B. Male gender
C. Age older than 60 years
D. Prior lumbar fusion
Discussion: D

17
Done by Dr.arafat alwadan my greating for u all

The patient has SI joint pain, which typically is felt in the buttock over the
joint and can cause referred pain into the leg. Physical examination findings
include tenderness over the joint (using the Fortin finger test); pain with
compression, distraction, and shear forces across the joint; pain with flexion,
abduction, and external rotation; and pain with the Gaenslen test. The
diagnosis is confirmed with an intra-articular injection. Prior lumbar fusion,
especially when involving more than 3 levels, has been associated with an
increased risk of SI joint pain. If nonsurgical care fails, minimally invasive
fusion of the SI joint can be an option that improves pain and function
outcomes better than continued nonsurgical care, according to several
studies. Polly and associates have shown that the amount of pain relief
following surgery does not correlate to the amount of relief following
injections. Pseudarthrosis rates have been reported to be about 5%.
Incidences of injury to the sacral nerve roots following surgery have been
reported.
Question 21 of 100

CLINICAL SITUATION

A 68-year-old man has a 6-month history of severe left buttock pain


extending down the back of the thigh. He has a past surgical history of fusion
from L3 to S1 performed 5 years earlier and a left total hip replacement
performed 2 years earlier. Recent radiographs of the hip are unremarkable.
On examination, he is noted to sit leaning towards the right side. The area
just below the left posterior superior iliac spine is tender to palpation. He has
a negative straight leg raise test but does have pain with flexion abduction
and external rotation of the left hip.

After the diagnosis is confirmed, initial nonsurgical treatment, which


included physical therapy and nonsteroidal anti-inflammatory drugs, fails.
When discussing surgery, you counsel the patient that
A. the degree of pain relief following surgery correlates to the degree of pain
improvement following injections.

18
Done by Dr.arafat alwadan my greating for u all

B. surgery does not result in as much reduced pain as does nonsurgical care.
C. a risk of nerve injury exists due to implant malpositioning.
D. the rate of pseudarthrosis exceeds 30%.
Discussion: C

The patient has SI joint pain, which typically is felt in the buttock over the
joint and can cause referred pain into the leg. Physical examination findings
include tenderness over the joint (using the Fortin finger test); pain with
compression, distraction, and shear forces across the joint; pain with flexion,
abduction, and external rotation; and pain with the Gaenslen test. The
diagnosis is confirmed with an intra-articular injection. Prior lumbar fusion,
especially when involving more than 3 levels, has been associated with an
increased risk of SI joint pain. If nonsurgical care fails, minimally invasive
fusion of the SI joint can be an option that improves pain and function
outcomes better than continued nonsurgical care, according to several
studies. Polly and associates have shown that the amount of pain relief
following surgery does not correlate to the amount of relief following
injections. Pseudarthrosis rates have been reported to be about 5%.
Incidences of injury to the sacral nerve roots following surgery have been
reported.
Question 22 of 100

Figures 1 through 3 show sagittal and axial MRIs and a radiograph from a
77-year-old woman with leg pain when standing and walking of 1 year
duration. The pain improves when she leans forward. She has been in physical
therapy, taken oral analgesics, and had epidural injections with minimal relief.
What is the best next step?

19
Done by Dr.arafat alwadan my greating for u all

A. Lateral interbody fusion


B. Laminectomy
C. Posterior spinal fusion with multilevel transforaminal lumbar interbody
fusion
D. Microdiskectomy
Discussion: B

The patient has lumbar stenosis of L2-3 and L3-4. She has no
spondylolisthesis or instability. For her condition, spinal fusion plays a
minimal role. She has no evidence of instability, and her condition can be
addressed through laminectomy only. No role exists for microdiskectomy,
because her disease results from a combination of ligamentum flavum
hypertrophy and facet hypertrophy.
Question 23 of 100

An elite football player sustains an on-field trauma. He has right C6 radicular


pain but has no weakness on physical examination or upper motor neuron
signs. MRI reveals a far-lateral right C5-6 disk herniation with no evidence of
cord compression. He would like to return to play as soon as possible. What
is best next step?
A. Surgery
B. Physical therapy
C. Repeat MRI
D. Return to play

20
Done by Dr.arafat alwadan my greating for u all

Discussion: B

The first line of treatment for a patient with radiculopathy without neurologic
deficits, regardless of profession, is nonsurgical treatment. If this approach
fails, an epidural steroid injection can be considered. If injection fails, surgery
is an option.
Question 24 of 100

When performing open transforaminal interbody fusion, the use of cortical


medial to lateral trajectory screws rather than traditional pedicle screws has
been shown to
A. increase fatigue pull-out strength.
B. reduce intraoperative blood loss.
C. improve reduction for patients with spondylolisthesis.
D. reduce the incidence of adjacent segment disease.
Discussion: B

Cortical medical to lateral screw fixation has been growing in popularity as


an alternative to traditional pedicle screw fixation. Lee and associates
compared 39 patients who underwent a transforaminal lumbar interbody
fusion performed with conventional screws with 40 patients who underwent
a transforaminal lumbar interbody fusion performed using cortical screws.
The authors showed equivalent fusion rates and similar Oswestry Disability
Index and Visual Analogue Scale (VAS) pain scores postoperatively. The
cortical screw group had less blood loss, shorter operating room times, and
shorter incision lengths. Similarly, Orita and associates reported equivalent
postoperative VAS pain scores compared with those from traditional
techniques. They also found lower fluoroscopy times and shorter incision
lengths. Ninomiya and associates showed equivalent radiographic outcomes,
including for the reduction of the spondylolisthesis and the restoration of
lumbar lordosis. Some controversy exists over the initial pull-out strength of
cortical screws; some studies show improved initial pull-out strength, and
others show it to be lower compared with that of traditional pedicle screws.

21
Done by Dr.arafat alwadan my greating for u all

Orita and associates showed that, after loading both types of the screws
through 100 cycles, traditional pedicle screws had statistically significantly
better fatigue strength. Cortical screws have not been shown to affect the
incidence of adjacent segment disease.
Question 25 of 100

CLINICAL SITUATION

Preoperative MRI images are shown from a 67-year-old woman with neck
pain, bilateral upper extremity paresthesias, progressively worsening
balance, several falls, and increasing problems in both hands with dropping
objects. Figure 1 is a sagittal view, Figure 2 is an axial cut at C2-3, Figure 3 is
an axial cut at C5-6, and Figure 4 is an axial cut at C6-7. The patient’s motor
strength is grade 4+ of 5 in the bilateral upper extremities.

What is the best next step?

A. Physical therapy
B. Cervical epidural steroid injection
C. Multilevel anterior cervical diskectomy and fusion
D. Posterior decompression and fusion
Discussion: D

The patient has degenerative changes with central and foraminal stenosis
from C2-7, including spinal cord compression at C2-3, C5-6, and C6-7. Her
progressively worsening balance, falls, and clumsiness are consistent with
cervical spondylotic myelopathy. Given the progressive nature of the
neurologic symptoms, surgery is recommended for patients with worsening

22
Done by Dr.arafat alwadan my greating for u all

symptoms. Physical therapy potentially could provide pain relief but would
not address the spinal cord compression. Cervical epidural steroid injection
is not recommended because of the increased neurologic risk in the setting
of substantial spinal cord compression. Given the multiple levels involved and
the extension to the C2-3 level, an anterior approach would increase the risk
of morbidity and pseudarthrosis compared with the posterior approach,
which would allow adequate decompression of the central and foraminal
stenosis.
Surgery for cervical myelopathy is performed to decompress the spinal cord,
stabilize the spine, and prevent further neurologic injury. Most patients
obtain considerable pain relief and some improvement in balance and
clumsiness, depending on the severity of the symptoms. Complete resolution
of all symptoms should not be expected.
The postoperative loss of strength in the bicep and deltoid are consistent
with C5 nerve palsy, which occurs in 0% to 30% of patients following cervical
decompression surgery. Commonly, symptoms occur several days
postoperatively. No findings suggested infection, and infection would not
lead to these symptoms. A transection of the nerve likely would have been
identified during surgery, would have caused immediate symptoms, and is
much less common than C5 nerve palsy. Most patients achieve near
complete resolution of symptoms within 6 months without further surgical
intervention, but the process can take more than 12 months in some cases.
No improvement is rare. Oral or injectable steroids and revision surgery are
not indicated.
Question 26 of 100

CLINICAL SITUATION

Preoperative MRI images are shown from a 67-year-old woman with neck
pain, bilateral upper extremity paresthesias, progressively worsening
balance, several falls, and increasing problems in both hands with dropping
objects. Figure 1 is a sagittal view, Figure 2 is an axial cut at C2-3, Figure 3 is
an axial cut at C5-6, and Figure 4 is an axial cut at C6-7. The patient’s motor
strength is grade 4+ of 5 in the bilateral upper extremities.

23
Done by Dr.arafat alwadan my greating for u all

What is the most likely outcome following treatment for this condition?

A. Pain relief only


B. Continued pain but restoration of balance
C. Halting of further progression with some balance improvement
D. Restoration of strength and balance, paresthesias and clumsiness to normal
Discussion: C

The patient has degenerative changes with central and foraminal stenosis
from C2-7, including spinal cord compression at C2-3, C5-6, and C6-7. Her
progressively worsening balance, falls, and clumsiness are consistent with
cervical spondylotic myelopathy. Given the progressive nature of the
neurologic symptoms, surgery is recommended for patients with worsening
symptoms. Physical therapy potentially could provide pain relief but would
not address the spinal cord compression. Cervical epidural steroid injection
is not recommended because of the increased neurologic risk in the setting
of substantial spinal cord compression. Given the multiple levels involved and
the extension to the C2-3 level, an anterior approach would increase the risk
of morbidity and pseudarthrosis compared with the posterior approach,
which would allow adequate decompression of the central and foraminal
stenosis.
Surgery for cervical myelopathy is performed to decompress the spinal cord,
stabilize the spine, and prevent further neurologic injury. Most patients
obtain considerable pain relief and some improvement in balance and
clumsiness, depending on the severity of the symptoms. Complete resolution
of all symptoms should not be expected.

24
Done by Dr.arafat alwadan my greating for u all

The postoperative loss of strength in the bicep and deltoid are consistent
with C5 nerve palsy, which occurs in 0% to 30% of patients following cervical
decompression surgery. Commonly, symptoms occur several days
postoperatively. No findings suggested infection, and infection would not
lead to these symptoms. A transection of the nerve likely would have been
identified during surgery, would have caused immediate symptoms, and is
much less common than C5 nerve palsy. Most patients achieve near
complete resolution of symptoms within 6 months without further surgical
intervention, but the process can take more than 12 months in some cases.
No improvement is rare. Oral or injectable steroids and revision surgery are
not indicated.
Question 27 of 100

CLINICAL SITUATION

Preoperative MRI images are shown from a 67-year-old woman with neck
pain, bilateral upper extremity paresthesias, progressively worsening
balance, several falls, and increasing problems in both hands with dropping
objects. Figure 1 is a sagittal view, Figure 2 is an axial cut at C2-3, Figure 3 is
an axial cut at C5-6, and Figure 4 is an axial cut at C6-7. The patient’s motor
strength is grade 4+ of 5 in the bilateral upper extremities.

Several days following treatment, weakness of grade 2 of 5 develops in the


right deltoid and biceps. What complication most likely caused this change?

A. Infection
B. C5 nerve palsy
C. Partial nerve transection

25
Done by Dr.arafat alwadan my greating for u all

D. Normal natural history


Discussion: B

The patient has degenerative changes with central and foraminal stenosis
from C2-7, including spinal cord compression at C2-3, C5-6, and C6-7. Her
progressively worsening balance, falls, and clumsiness are consistent with
cervical spondylotic myelopathy. Given the progressive nature of the
neurologic symptoms, surgery is recommended for patients with worsening
symptoms. Physical therapy potentially could provide pain relief but would
not address the spinal cord compression. Cervical epidural steroid injection
is not recommended because of the increased neurologic risk in the setting
of substantial spinal cord compression. Given the multiple levels involved and
the extension to the C2-3 level, an anterior approach would increase the risk
of morbidity and pseudarthrosis compared with the posterior approach,
which would allow adequate decompression of the central and foraminal
stenosis.
Surgery for cervical myelopathy is performed to decompress the spinal cord,
stabilize the spine, and prevent further neurologic injury. Most patients
obtain considerable pain relief and some improvement in balance and
clumsiness, depending on the severity of the symptoms. Complete resolution
of all symptoms should not be expected.
The postoperative loss of strength in the bicep and deltoid are consistent
with C5 nerve palsy, which occurs in 0% to 30% of patients following cervical
decompression surgery. Commonly, symptoms occur several days
postoperatively. No findings suggested infection, and infection would not
lead to these symptoms. A transection of the nerve likely would have been
identified during surgery, would have caused immediate symptoms, and is
much less common than C5 nerve palsy. Most patients achieve near
complete resolution of symptoms within 6 months without further surgical
intervention, but the process can take more than 12 months in some cases.
No improvement is rare. Oral or injectable steroids and revision surgery are
not indicated.

26
Done by Dr.arafat alwadan my greating for u all

Question 28 of 100

CLINICAL SITUATION

Preoperative MRI images are shown from a 67-year-old woman with neck
pain, bilateral upper extremity paresthesias, progressively worsening
balance, several falls, and increasing problems in both hands with dropping
objects. Figure 1 is a sagittal view, Figure 2 is an axial cut at C2-3, Figure 3 is
an axial cut at C5-6, and Figure 4 is an axial cut at C6-7. The patient’s motor
strength is grade 4+ of 5 in the bilateral upper extremities.

What is the most appropriate treatment for this complication?

A. Observation and reassurance


B. High-dose oral steroids
C. Cervical epidural steroid injection
D. Revision surgery
Discussion: A

The patient has degenerative changes with central and foraminal stenosis
from C2-7, including spinal cord compression at C2-3, C5-6, and C6-7. Her
progressively worsening balance, falls, and clumsiness are consistent with
cervical spondylotic myelopathy. Given the progressive nature of the
neurologic symptoms, surgery is recommended for patients with worsening
symptoms. Physical therapy potentially could provide pain relief but would
not address the spinal cord compression. Cervical epidural steroid injection
is not recommended because of the increased neurologic risk in the setting
of substantial spinal cord compression. Given the multiple levels involved and
the extension to the C2-3 level, an anterior approach would increase the risk

27
Done by Dr.arafat alwadan my greating for u all

of morbidity and pseudarthrosis compared with the posterior approach,


which would allow adequate decompression of the central and foraminal
stenosis.
Surgery for cervical myelopathy is performed to decompress the spinal cord,
stabilize the spine, and prevent further neurologic injury. Most patients
obtain considerable pain relief and some improvement in balance and
clumsiness, depending on the severity of the symptoms. Complete resolution
of all symptoms should not be expected.
The postoperative loss of strength in the bicep and deltoid are consistent
with C5 nerve palsy, which occurs in 0% to 30% of patients following cervical
decompression surgery. Commonly, symptoms occur several days
postoperatively. No findings suggested infection, and infection would not
lead to these symptoms. A transection of the nerve likely would have been
identified during surgery, would have caused immediate symptoms, and is
much less common than C5 nerve palsy. Most patients achieve near
complete resolution of symptoms within 6 months without further surgical
intervention, but the process can take more than 12 months in some cases.
No improvement is rare. Oral or injectable steroids and revision surgery are
not indicated.
Question 29 of 100

CLINICAL SITUATION

Preoperative MRI images are shown from a 67-year-old woman with neck
pain, bilateral upper extremity paresthesias, progressively worsening
balance, several falls, and increasing problems in both hands with dropping
objects. Figure 1 is a sagittal view, Figure 2 is an axial cut at C2-3, Figure 3 is
an axial cut at C5-6, and Figure 4 is an axial cut at C6-7. The patient’s motor
strength is grade 4+ of 5 in the bilateral upper extremities.

What is the most likely outcome of this complication?

28
Done by Dr.arafat alwadan my greating for u all

A. Complete recovery within 1 week


B. Near complete recovery within 12 months
C. 1 motor grade recovery within 12 months
D. Permanent condition with no recovery
Discussion: B

The patient has degenerative changes with central and foraminal stenosis
from C2-7, including spinal cord compression at C2-3, C5-6, and C6-7. Her
progressively worsening balance, falls, and clumsiness are consistent with
cervical spondylotic myelopathy. Given the progressive nature of the
neurologic symptoms, surgery is recommended for patients with worsening
symptoms. Physical therapy potentially could provide pain relief but would
not address the spinal cord compression. Cervical epidural steroid injection
is not recommended because of the increased neurologic risk in the setting
of substantial spinal cord compression. Given the multiple levels involved and
the extension to the C2-3 level, an anterior approach would increase the risk
of morbidity and pseudarthrosis compared with the posterior approach,
which would allow adequate decompression of the central and foraminal
stenosis.
Surgery for cervical myelopathy is performed to decompress the spinal cord,
stabilize the spine, and prevent further neurologic injury. Most patients
obtain considerable pain relief and some improvement in balance and
clumsiness, depending on the severity of the symptoms. Complete resolution
of all symptoms should not be expected.
The postoperative loss of strength in the bicep and deltoid are consistent
with C5 nerve palsy, which occurs in 0% to 30% of patients following cervical

29
Done by Dr.arafat alwadan my greating for u all

decompression surgery. Commonly, symptoms occur several days


postoperatively. No findings suggested infection, and infection would not
lead to these symptoms. A transection of the nerve likely would have been
identified during surgery, would have caused immediate symptoms, and is
much less common than C5 nerve palsy. Most patients achieve near
complete resolution of symptoms within 6 months without further surgical
intervention, but the process can take more than 12 months in some cases.
No improvement is rare. Oral or injectable steroids and revision surgery are
not indicated.
Question 30 of 100

Figure 1 depicts the cervical MRI from a 40-year-old woman with a 1-month
history of neck pain, neck stiffness, and electric-like right arm pain with
certain neck movements. She has tried anti-inflammatory medication for the
pain. On physical examination, she has a normal tandem gait, her motor and
sensory functions are intact, and she has normal reflexes. She displays a
positive Spurling sign. The patient states that she can relieve her symptoms
temporarily by raising her right arm over her head. What is the best next
step?

A. Referral to physical therapy


B. Prescription for immediate release opioids
C. Scheduling of an elective anterior cervical discectomy and fusion
D. Scheduling of an elective cervical total disk arthroplasty

30
Done by Dr.arafat alwadan my greating for u all

Discussion: A

This patient has a cervical disk herniation with symptomatic radiculopathy.


The herniation is still in the acute phase, and the patient gives no account of
clinical progression or worsening. Generally, the natural history of cervical
radiculopathy is favorable, with resolution in most cases. Nonsurgical
management remains a reasonable treatment option at this point. The use
of opioid medications carries the risk of addiction or abuse; therefore,
prescribing opioids at this time is not desirable. Because she does not have
myelopathy or rapidly progressive neurologic symptoms, surgical treatment
also is not advisable at this time. The onset of symptoms remains acute at
only 4 weeks.
Question 31 of 100

An elite baseball player sustains a twisting injury to his back. He was found
to have a lumbar disk herniation. Subsequently, 6 weeks of nonsurgical
treatment failed, and the patient underwent a microdiskectomy. When
counseling the patient, what would be the best assessment of the
expectations for return to play and performance, as well as career?
A. No return to play
B. Return to play but at less than 25% of the preoperative baseline level
C. Return to play at more than 25% but less than 50% of the preoperative
baseline level
D. Return to play at more than 50% but less than 100% of the preoperative
baseline level
Discussion: D

A high proportion of elite football, baseball, and hockey athletes (50% to


100%), return to play after a lumbar microdiskectomy. In general, baseball
players have the longest average career length after surgery, compared with
football players and hockey players.

31
Done by Dr.arafat alwadan my greating for u all

Question 32 of 100

A 63-year-old man with a prior L4-S1 laminectomy and posterior fusion with
instrumentation performed 10 years earlier reports buttock and leg pain. He
stands with his hips and knees flexed. He has a degenerative grade 1
spondylolisthesis at the L3-4 level. On flexion views, the spondylolisthesis
worsens by 5 mm and reduces on extension. His pelvic incidence (PI) is 45
degrees and his lumbar lordosis (LL) is 25 degrees. Other than a three-
column osteotomy, what is the best surgical option to address the patient’s
sagittal imbalance?
A. Posterior column osteotomy (Smith Petersen or Ponte) at L3 and extension
of the posterior fusion to L3
B. Transforaminal interbody fusion with bilateral facetectomy and extension of
the posterior fusion to L3
C. Lateral interbody fusion with a hyperlordotic cage and anterior column
release with extension of the posterior fusion to L3
D. Posterior pedicle screw fixation and extension of fusion to T10
Discussion: C

In addition to the patient’s stenosis and spondylolisthesis, a spinopelvis


imbalance is present. A number of equations relate LL and PI, but the formula
LL=PI +/- 9 degrees is used commonly. In this case, the patient needs at least
15degrees of correction. The posterior based osteotomies can provide 3
degrees to 5degrees of correction per level. A transforaminal lumbar
interbody fusion with complete fasciotomy has been shown to produce 8
degrees of correction. Posterior fusion alone is not likely to result in any
correction. Releasing the anterior longitudinal ligament and using a
hyperlordotic graft while performing a lateral interbody fusion has been
shown to result in 10 degrees to 15 degrees of correction. The addition of
posterior column osteotomies has been shown to increase this correction to
22degrees. Akbarnia and associates reported a high rate of complications
when performing an anterior column release, but most occurred at the L4-5
level. Subsequent studies by Murray and associates and Wang and associates

32
Done by Dr.arafat alwadan my greating for u all

have reported lower rates of complications, especially when the release is


performed above the L4-5 level. Extension of the construct to T10 would
require fusing more levels and might not correct the sagittal alignment
without the addition of multiple osteotomies.
Question 33 of 100

CLINICAL SITUATION

A 77-year-old man has difficulty walking and reports that he has been
dropping things frequently. He states that his symptoms have gotten worse
over the last 12 months, and he now has trouble buttoning his shirts. He has
a history of ulcerative colitis and diabetes mellitus, as well as coronary artery
disease, for which he takes aspirin. He continues to smoke daily, however. His
MRI reveals well-maintained lordosis and multilevel, severe cervical stenosis.
His modified Japanese Orthopedic Association (mJOA) score is 11.

A posterior cervical decompression and instrumented fusion is planned to


relieve this patient’s symptoms. What would you counsel the patient is the
most common complication associated with this approach in this case?
A. Blindness
B. Worsening myelopathy
C. Wound infection
D. Dural tear
Discussion: C

A large prospective study analyzed the quality of life after surgery compared
with preoperative levels for patients with cervical myelopathy. Fehling and
associates found that the quality of life improved after surgery even when
patients were analyzed based on their preoperative mJOA scores (mild,
medium, or severe myelopathy). C5 nerve root palsy occurs relatively
commonly after cervical decompression, with an incidence of about 5% (Lim
and associates). One study by Lee and associates reported the rate to be
28.9% after posterior decompression and fusion. Most patients (92%)
improve by 2 years after surgery. Risk factors for persistent symptoms

33
Done by Dr.arafat alwadan my greating for u all

included a motor grade of less than or equal to 2 of 5, multisegment


dysfunction involving more than C5, and loss of sensation with pain (Lim and
associates). Infection is another complication that is more common with a
posterior cervical spine approach, and the risk of infection is increased by the
presence of diabetes. A recent prospective study by Tetreault and associates
revealed an increased risk of perioperative complications in patients with
diabetes mellitus, ossification of the posterior longitudinal ligament, longer
surgical duration, and more medical comorbidities. Another large, multi-
institutional study by Tetreault and associates sought to predict which
patients would improve after surgical decompression of cervical spondylotic
myelopathy. The authors found that certain patient factors were associated
with a poorer clinical outcome, including older age, a worse baseline
myelopathy score, impaired gait, more medical comorbidities, smoking, and
a longer duration of symptoms.
Question 34 of 100

CLINICAL SITUATION

A 77-year-old man has difficulty walking and reports that he has been
dropping things frequently. He states that his symptoms have gotten worse
over the last 12 months, and he now has trouble buttoning his shirts. He has
a history of ulcerative colitis and diabetes mellitus, as well as coronary artery
disease, for which he takes aspirin. He continues to smoke daily, however. His
MRI reveals well-maintained lordosis and multilevel, severe cervical stenosis.
His modified Japanese Orthopedic Association (mJOA) score is 11.

In patients with severe cervical spondylotic myelopathy (mJOA score lower


than 12), how is the quality of life expected to change following
decompressive surgery?
A. Worse than the preoperative status
B. Stable compared with the preoperative status
C. Improved, but not statistically significantly
D. Statistically significant improvement

34
Done by Dr.arafat alwadan my greating for u all

Discussion: D

A large prospective study analyzed the quality of life after surgery compared
with preoperative levels for patients with cervical myelopathy. Fehling and
associates found that the quality of life improved after surgery even when
patients were analyzed based on their preoperative mJOA scores (mild,
medium, or severe myelopathy). C5 nerve root palsy occurs relatively
commonly after cervical decompression, with an incidence of about 5% (Lim
and associates). One study by Lee and associates reported the rate to be
28.9% after posterior decompression and fusion. Most patients (92%)
improve by 2 years after surgery. Risk factors for persistent symptoms
included a motor grade of less than or equal to 2 of 5, multisegment
dysfunction involving more than C5, and loss of sensation with pain (Lim and
associates). Infection is another complication that is more common with a
posterior cervical spine approach, and the risk of infection is increased by the
presence of diabetes. A recent prospective study by Tetreault and associates
revealed an increased risk of perioperative complications in patients with
diabetes mellitus, ossification of the posterior longitudinal ligament, longer
surgical duration, and more medical comorbidities. Another large, multi-
institutional study by Tetreault and associates sought to predict which
patients would improve after surgical decompression of cervical spondylotic
myelopathy. The authors found that certain patient factors were associated
with a poorer clinical outcome, including older age, a worse baseline
myelopathy score, impaired gait, more medical comorbidities, smoking, and
a longer duration of symptoms.
Question 35 of 100

CLINICAL SITUATION

A 77-year-old man has difficulty walking and reports that he has been
dropping things frequently. He states that his symptoms have gotten worse
over the last 12 months, and he now has trouble buttoning his shirts. He has
a history of ulcerative colitis and diabetes mellitus, as well as coronary artery

35
Done by Dr.arafat alwadan my greating for u all

disease, for which he takes aspirin. He continues to smoke daily, however. His
MRI reveals well-maintained lordosis and multilevel, severe cervical stenosis.
His modified Japanese Orthopedic Association (mJOA) score is 11.

The patient undergoes posterior cervical laminectomy and instrumented


fusion. On postoperative day 2 new, painless right deltoid weakness develops
(grade 3 of 5). What is the most likely outcome of this new C5 nerve root
palsy?
A. Minimal improvement by 2 years
B. Full improvement by 2 years
C. Full improvement by 3 months
D. Worsening strength over time
Discussion: B

A large prospective study analyzed the quality of life after surgery compared
with preoperative levels for patients with cervical myelopathy. Fehling and
associates found that the quality of life improved after surgery even when
patients were analyzed based on their preoperative mJOA scores (mild,
medium, or severe myelopathy). C5 nerve root palsy occurs relatively
commonly after cervical decompression, with an incidence of about 5% (Lim
and associates). One study by Lee and associates reported the rate to be
28.9% after posterior decompression and fusion. Most patients (92%)
improve by 2 years after surgery. Risk factors for persistent symptoms
included a motor grade of less than or equal to 2 of 5, multisegment
dysfunction involving more than C5, and loss of sensation with pain (Lim and
associates). Infection is another complication that is more common with a
posterior cervical spine approach, and the risk of infection is increased by the
presence of diabetes. A recent prospective study by Tetreault and associates
revealed an increased risk of perioperative complications in patients with
diabetes mellitus, ossification of the posterior longitudinal ligament, longer
surgical duration, and more medical comorbidities. Another large, multi-
institutional study by Tetreault and associates sought to predict which
patients would improve after surgical decompression of cervical spondylotic

36
Done by Dr.arafat alwadan my greating for u all

myelopathy. The authors found that certain patient factors were associated
with a poorer clinical outcome, including older age, a worse baseline
myelopathy score, impaired gait, more medical comorbidities, smoking, and
a longer duration of symptoms.
Question 36 of 100

CLINICAL SITUATION

A 77-year-old man has difficulty walking and reports that he has been
dropping things frequently. He states that his symptoms have gotten worse
over the last 12 months, and he now has trouble buttoning his shirts. He has
a history of ulcerative colitis and diabetes mellitus, as well as coronary artery
disease, for which he takes aspirin. He continues to smoke daily, however. His
MRI reveals well-maintained lordosis and multilevel, severe cervical stenosis.
His modified Japanese Orthopedic Association (mJOA) score is 11.

What is the probability that this patient’s myelopathy will improve to mild
(mJOA score higher than 16) postoperatively?
A. More than 80%
B. Between 50% and 80%
C. Between 20% and 50%
D. Less than 20%
Discussion: D

A large prospective study analyzed the quality of life after surgery compared
with preoperative levels for patients with cervical myelopathy. Fehling and
associates found that the quality of life improved after surgery even when
patients were analyzed based on their preoperative mJOA scores (mild,
medium, or severe myelopathy). C5 nerve root palsy occurs relatively
commonly after cervical decompression, with an incidence of about 5% (Lim
and associates). One study by Lee and associates reported the rate to be
28.9% after posterior decompression and fusion. Most patients (92%)
improve by 2 years after surgery. Risk factors for persistent symptoms
included a motor grade of less than or equal to 2 of 5, multisegment

37
Done by Dr.arafat alwadan my greating for u all

dysfunction involving more than C5, and loss of sensation with pain (Lim and
associates). Infection is another complication that is more common with a
posterior cervical spine approach, and the risk of infection is increased by the
presence of diabetes. A recent prospective study by Tetreault and associates
revealed an increased risk of perioperative complications in patients with
diabetes mellitus, ossification of the posterior longitudinal ligament, longer
surgical duration, and more medical comorbidities. Another large, multi-
institutional study by Tetreault and associates sought to predict which
patients would improve after surgical decompression of cervical spondylotic
myelopathy. The authors found that certain patient factors were associated
with a poorer clinical outcome, including older age, a worse baseline
myelopathy score, impaired gait, more medical comorbidities, smoking, and
a longer duration of symptoms.
Question 37 of 100

Figures 1 through 3 show the radiograph and CT images of a 68-year-old


woman who sustained a cervical injury after tripping over her cat. She has
mild facial trauma, which includes a broken upper incisor and a nonsurgical
nasal fracture. She is neurologically intact. Her past medical history is
consistent with obstructive sleep apnea, non–insulin-dependent diabetes
mellitus (hemoglobin A1c level of 9.0), and morbid obesity, with a body mass
index of 40. What is the preferred treatment for this patient?

A. Management with a hard collar


B. Halo vest immobilization
C. Posterior C1-2 arthrodesis
D. Anterior odontoid screw fixation
Discussion: C

38
Done by Dr.arafat alwadan my greating for u all

Posterior C1-2 fusion with instrumentation provides stability and pain relief
with excellent clinical outcomes despite the loss of C1-2 motion. Hard collar
immobilization and halo vest immobilization both carry a substantial risk of
nonunion in this patient because of her age, fracture displacement, residual
fracture gap, and medical condition. Anterior odontoid screw fixation
theoretically preserves C1-2 motion. In this case, the fracture is not reduced.
Concentric reduction is a requisite for osteosynthesis of the odontoid. Her
body habitus also may not allow anterior odontoid fixation.
Question 38 of 100

Figure 1 depicts a T2-weighted MRI from a 30-year-old man with 6 months


of progressive low back pain. The best next step should be staging studies
followed by

A. radiation and surgery.


B. biopsy.
C. posterior resection.
D. anterior/posterior resection.
Discussion: B

It is important to stage this patient to assess whether he has metastatic


disease. It is most important to obtain a biopsy before beginning any
treatment, however. The patient’s treatment would be vastly different if this
mass was revealed to be an osteosarcoma, lymphoma, or chordoma.

39
Done by Dr.arafat alwadan my greating for u all

Question 39 of 100

A 57-year-old man with metastatic lung cancer undergoes revision surgery


for decompression and stabilization of the C7 lesion. He has had prior
radiation to the surgical region, as well as chemotherapy for the lung cancer.
A wound dehiscence develops 6 weeks postoperatively. The intraoperative
cultures are negative. What preventive option may have reduced his chance
of wound complications?
A. Incisional vacuum-assisted closure
B. Total parenteral nutrition
C. Intrawound vancomycin powder
D. Initial soft-tissue closure by plastic surgery
Discussion: D

In the setting of revision spine tumor surgery and in previously radiated


wounds, having a plastic surgeon perform the soft-tissue reconstruction and
closure has been associated with reduced rates of wound complications.
Although poor nutrition is a risk factor for delayed wound healing, no
evidence supports total parenteral nutrition. Incisional vacuum-assisted
closure has not been studied in the setting of spine tumor surgery.
Intrawound vancomycin powder has been shown to reduce surgical site
infections in the setting of spine tumor surgery but has no effect on wound
dehiscence.
Question 40 of 100

A 45-year-old man with a single C5-6 disk herniation presents for surgical
consultation. He has no history of inflammatory disease, infection, or chronic
illness. He has not had previous neck surgery. His radiographs show no
instability and no evidence of severe facet arthrosis. He is very active. What
can you tell him about deciding to proceed with total disk arthroplasty (TDA)
rather than anterior cervical diskectomy and fusion (ACDF)?

40
Done by Dr.arafat alwadan my greating for u all

A. TDA remains too new a procedure to be compared adequately with the gold
standard, ACDF.
B. TDA is not Food and Drug Administration (FDA) approved, and its use would
be as an off-label device.
C. In prospective trials, TDA has been found to be superior to ACDF in clinical
outcomes for one-level and two-level disease.
D. Both procedures have good clinical outcomes. Some newer studies show
lower reoperation rates and a lower incidence of adjacent segment
degeneration with TDA.
Discussion: D

In several studies, it has been shown that TDA compares very favorably with
ACDF in clinical outcomes as scored by the Neck Disability Index, the Visual
Analog Scale, the 16-Item and 32-Item Short-Form Health Surveys, and by
safety. More recent literature has shown that TDA also has lower rates of
reoperation and adjacent segment degeneration than ACDF, in effect making
TDA a viable alternative to ACDF in appropriate cases. Both 5-year and 10-
year survivorship studies of TDA have appeared in publication. The FDA has
approved several cervical TDA devices, including the BRYAN Cervical Artificial
Disc, the ProDisc-C Total Disc Replacement device, the SECURE-C Artificial
Cervical Disc, the Prestige LP Cervical Artificial Disc, and the Mobi-C Artificial
Cervical Disc.
Question 41 of 100

A 55-year-old man with a history of diabetes undergoes multilevel lumbar


laminectomy for spinal stenosis. He is given intravenous (IV) antibiotics
before the incision and again for the first 24 hours of recovery. A lumbar
drain is placed and then removed 72 hours after surgery. What factor is most
associated with the development of a wound infection in this scenario?
A. Lack of antibiotic coverage while the drain is present
B. Retention of the Foley catheter for longer than 24 hours
C. Presence of diabetes mellitus
D. Placement of a drain

41
Done by Dr.arafat alwadan my greating for u all

Discussion: C

The use of drains in spinal surgery remains controversial. Prospective studies


have shown that the use of drainage devices does not increase the rate of
infection, however (Brown and associates). Also, some physicians continue to
deliver antibiotics when the drain is present. A recent prospective
randomized study demonstrated no difference in postoperative infections
when comparing the use of IV antibiotics for 24 hours with the use of IV
antibiotics for the duration of the drain placement. The study concluded that
the use of IV antibiotics should include the standard preoperative dose
followed by 24 hours of antibiotics and then discontinuation of the
antibiotics (Takemoto and associates). Retention of Foley catheters for longer
than 24 hours has been associated with an increased risk of urinary tract
infection but has not been associated with an increased risk of wound
infection.
Question 42 of 100

What pelvic parameter is fixed as an adult?


A. Sacral slope (SS)
B. Pelvic incidence (PI)
C. Pelvic tilt (PT)
D. Slip angle
Discussion: B

PI is a fixed measurement in an adult. SS and PT can change as adult


deformity worsens. PI is the angle calculated by a line drawn from the center
of the femoral head to the midpoint of the sacral end plate and the line
perpendicular to the center of the sacral superior endplate. This angle is fixed
in adults. PT is the angle calculated by a vertical reference line from the center
of the femoral head and the midpoint of the sacral endplate. SS is defined as
the angle calculated by a horizontal reference line and the sacral endplate
line. PI = PT + SS. Lumbar lordosis = PI (+/- 9 degrees).

42
Done by Dr.arafat alwadan my greating for u all

Question 43 of 100

Figures 1 and 2 depict the MRIs of a 39-year-old woman with severe right
leg pain and mild back pain. Symptoms have been present for 3 months. She
has tried physical therapy and received transforaminal injections and oral
pain medications with minimal relief. She is offered a lumbar
microdiskectomy. During the procedure, she sustains an incidental durotomy
which is repaired primarily. She is informed that the rate of reoperation
following lumbar microdiskectomy is

A. 5%.
B. 15%.
C. 25%.
D. 35%.
Discussion: B

Recurrent disk herniation and reoperations are common complications


following lumbar microdiskectomy. In the Spine Patient Outcomes Research
Trial, the reoperation rate following diskectomy was 15% at 8-year follow-up.
In this same cohort, no long-term impact of incidental durotomies was
sustained during these procedures.
Question 44 of 100

CLINICAL SITUATION

Figures 1 and 2 are the sagittal and axial MRI images at the L4-5 level from a
patient with intractable left leg pain, paresthesias, and weakness of grade 3

43
Done by Dr.arafat alwadan my greating for u all

of 5. Physical therapy has failed, and the patient feels he is becoming


progressively weaker.

Which nerve root is being affected?

A. L3
B. L4
C. L5
D. S1
Discussion: C

The MRI images show a left L4-5 posterolateral disk herniation compressing
the left L5 nerve root. Posterolateral disk herniations in the lumbar spine
affect the traversing nerve roots; therefore, at the L4-5 level, the traversing
L5 nerve would be affected. Far-lateral disk herniations in the lumbar spine
affect the exiting nerve roots, so a far-lateral herniation at L5-S1 would affect
the L5 nerve root also. The motor innervation for the lumbar nerve roots are
L2: hip flexion, L3: knee extension, L4: ankle dorsiflexion, L5: long toe
extension, and S1: ankle plantar flexion. The sensory innervation for the
lumbar nerve roots are L3: medial thigh, L4: anterolateral thigh and medial
calf, L5: anterolateral calf, and S1: lateral foot. Nonsurgical treatment,
including medications and physical therapy, has failed, and the patient has
noticed a progressive deficit confirmed by weakness of grade 3 of 5 on
examination. Surgery is indicated because of the patient’s progressive
weakness and the failure of nonsurgical care. A fusion is not indicated
without evidence of instability.
Question 45 of 100

CLINICAL SITUATION

44
Done by Dr.arafat alwadan my greating for u all

Figures 1 and 2 are the sagittal and axial MRI images at the L4-5 level from a
patient with intractable left leg pain, paresthesias, and weakness of grade 3
of 5. Physical therapy has failed, and the patient feels he is becoming
progressively weaker.

What other disk herniation location also could affect the same nerve root?

A. Far-lateral L4-5
B. Far-lateral L5-S1
C. Posterolateral L3-4
D. Posterolateral L5-S1
Discussion: B

The MRI images show a left L4-5 posterolateral disk herniation compressing
the left L5 nerve root. Posterolateral disk herniations in the lumbar spine
affect the traversing nerve roots; therefore, at the L4-5 level, the traversing
L5 nerve would be affected. Far-lateral disk herniations in the lumbar spine
affect the exiting nerve roots, so a far-lateral herniation at L5-S1 would affect
the L5 nerve root also. The motor innervation for the lumbar nerve roots are
L2: hip flexion, L3: knee extension, L4: ankle dorsiflexion, L5: long toe
extension, and S1: ankle plantar flexion. The sensory innervation for the
lumbar nerve roots are L3: medial thigh, L4: anterolateral thigh and medial
calf, L5: anterolateral calf, and S1: lateral foot. Nonsurgical treatment,
including medications and physical therapy, has failed, and the patient has
noticed a progressive deficit confirmed by weakness of grade 3 of 5 on
examination. Surgery is indicated because of the patient’s progressive
weakness and the failure of nonsurgical care. A fusion is not indicated
without evidence of instability.

45
Done by Dr.arafat alwadan my greating for u all

Question 46 of 100

CLINICAL SITUATION

Figures 1 and 2 are the sagittal and axial MRI images at the L4-5 level from a
patient with intractable left leg pain, paresthesias, and weakness of grade 3
of 5. Physical therapy has failed, and the patient feels he is becoming
progressively weaker.

Which muscle function most likely would be affected in this patient?

A. Hip flexion
B. Ankle dorsiflexion
C. Long toe extension
D. Ankle plantar flexion
Discussion: C

The MRI images show a left L4-5 posterolateral disk herniation compressing
the left L5 nerve root. Posterolateral disk herniations in the lumbar spine
affect the traversing nerve roots; therefore, at the L4-5 level, the traversing
L5 nerve would be affected. Far-lateral disk herniations in the lumbar spine
affect the exiting nerve roots, so a far-lateral herniation at L5-S1 would affect
the L5 nerve root also. The motor innervation for the lumbar nerve roots are
L2: hip flexion, L3: knee extension, L4: ankle dorsiflexion, L5: long toe
extension, and S1: ankle plantar flexion. The sensory innervation for the
lumbar nerve roots are L3: medial thigh, L4: anterolateral thigh and medial
calf, L5: anterolateral calf, and S1: lateral foot. Nonsurgical treatment,
including medications and physical therapy, has failed, and the patient has
noticed a progressive deficit confirmed by weakness of grade 3 of 5 on

46
Done by Dr.arafat alwadan my greating for u all

examination. Surgery is indicated because of the patient’s progressive


weakness and the failure of nonsurgical care. A fusion is not indicated
without evidence of instability.
Question 47 of 100

CLINICAL SITUATION

Figures 1 and 2 are the sagittal and axial MRI images at the L4-5 level from a
patient with intractable left leg pain, paresthesias, and weakness of grade 3
of 5. Physical therapy has failed, and the patient feels he is becoming
progressively weaker.

Which location most likely would be affected by the paresthesias?

A. Medial thigh
B. Lateral foot
C. Medial calf
D. Anterolateral calf
Discussion: D

The MRI images show a left L4-5 posterolateral disk herniation compressing
the left L5 nerve root. Posterolateral disk herniations in the lumbar spine
affect the traversing nerve roots; therefore, at the L4-5 level, the traversing
L5 nerve would be affected. Far-lateral disk herniations in the lumbar spine
affect the exiting nerve roots, so a far-lateral herniation at L5-S1 would affect
the L5 nerve root also. The motor innervation for the lumbar nerve roots are
L2: hip flexion, L3: knee extension, L4: ankle dorsiflexion, L5: long toe
extension, and S1: ankle plantar flexion. The sensory innervation for the
lumbar nerve roots are L3: medial thigh, L4: anterolateral thigh and medial

47
Done by Dr.arafat alwadan my greating for u all

calf, L5: anterolateral calf, and S1: lateral foot. Nonsurgical treatment,
including medications and physical therapy, has failed, and the patient has
noticed a progressive deficit confirmed by weakness of grade 3 of 5 on
examination. Surgery is indicated because of the patient’s progressive
weakness and the failure of nonsurgical care. A fusion is not indicated
without evidence of instability.
Question 48 of 100

CLINICAL SITUATION

Figures 1 and 2 are the sagittal and axial MRI images at the L4-5 level from a
patient with intractable left leg pain, paresthesias, and weakness of grade 3
of 5. Physical therapy has failed, and the patient feels he is becoming
progressively weaker.

What is the best next step?

A. Observation
B. L4-5 epidural steroid injection
C. L4-5 microdiskectomy
D. L4-5 transforaminal interbody fusion
Discussion: C

The MRI images show a left L4-5 posterolateral disk herniation compressing
the left L5 nerve root. Posterolateral disk herniations in the lumbar spine
affect the traversing nerve roots; therefore, at the L4-5 level, the traversing
L5 nerve would be affected. Far-lateral disk herniations in the lumbar spine
affect the exiting nerve roots, so a far-lateral herniation at L5-S1 would affect
the L5 nerve root also. The motor innervation for the lumbar nerve roots are

48
Done by Dr.arafat alwadan my greating for u all

L2: hip flexion, L3: knee extension, L4: ankle dorsiflexion, L5: long toe
extension, and S1: ankle plantar flexion. The sensory innervation for the
lumbar nerve roots are L3: medial thigh, L4: anterolateral thigh and medial
calf, L5: anterolateral calf, and S1: lateral foot. Nonsurgical treatment,
including medications and physical therapy, has failed, and the patient has
noticed a progressive deficit confirmed by weakness of grade 3 of 5 on
examination. Surgery is indicated because of the patient’s progressive
weakness and the failure of nonsurgical care. A fusion is not indicated
without evidence of instability.
Question 49 of 100

When performing an anterior cervical decompression and fusion, the


placement of a steroid medication anterior to the plate at the completion of
the case has been shown to
A. increase the risk of infection.
B. reduce postoperative pain.
C. reduce postoperative dysphagia.
D. increase the risk of pseudarthrosis.
Discussion: C

Multiple studies have evaluated the use of retropharyngeally placed steroid


prior to wound closure during anterior cervical decompression and fusion
surgery. They have reported statistically significantly reduced rates of
dysphagia. These studies have not shown the steroids to have had any effect
on postoperative pain or the rate of infection. Jeyamohan and associates
suggested that a delay in fusion may occur but did not find any difference in
overall fusion rates. In larger studies, Cancienne and associates and Koreckij
and associates did not find any difference in fusion rates between the steroid
and control groups following the use of retropharyngeal steroids.

49
Done by Dr.arafat alwadan my greating for u all

Question 50 of 100

Figure 1 is an axial MRI at the L4-5 level obtained from a 62-year-old man
with a 6-month history of severe right leg pain and weakness in the ankle
dorsiflexors. He has numbness along the medial ankle and dorsolateral
aspect of his foot. The structure identified by the arrow is compressing what
neural structure?

A. Traversing right L4 nerve root


B. Exiting right L5 nerve root
C. Exiting left L4 nerve root
D. Exiting right L4 nerve root
Discussion: D

This patient has the clinical symptoms of a right L4 lumbar radiculopathy. The
MRI taken at L4-5 shows a far-lateral/foraminal disk herniation. This disk
herniation would compress the exiting L4 nerve root along with its dorsal
root ganglion. The traversing right L4 nerve root would be seen best in an
axial MRI at the L3-4 level. The exiting right L5 nerve root would be seen best
in an axial MRI at the L5-S1 level. The disk herniation in question is right
sided. The left neuroforamen is free in the axial MRI.
Question 51 of 100

On examination, a clinician finds that a patient has difficulty with grip and
release, loss of motor strength, sensory changes, intrinsic wasting, the finger
escape sign, and spasticity. These findings are best described as
A. myelopathic hand.

50
Done by Dr.arafat alwadan my greating for u all

B. rheumatoid arthritis of the hand.


C. carpal tunnel hand.
D. brachial plexopathy.
Discussion: A

Myelopathic hand is a term used to describe a patient with myelopathy and


myelopathic findings in the hand. Typical myelopathic symptoms include
upper motor findings, including difficulty with hand dexterity, hyperreflexia,
a positive Hoffman sign, spasticity, a positive Romberg sign, and gait
changes/ataxia.
Question 52 of 100

Figure 1 is the MRI from a 67-year-old man with severe neck pain 1 week
following dental extraction. He has a history of poorly controlled type 2
diabetes mellitus. On examination, he is found to have grade 4 of 5 strength
in the bilateral lower extremities. He is febrile and has an elevated erythrocyte
sedimentation rate and an elevated C-reactive protein level. His MRI
reveals an epidural abscess. What is the best next step?

A. Intravenous antibiotics
B. Observation and intravenous antibiotics
C. Surgical decompression
D. Interventional radiology drainage
Discussion: C

51
Done by Dr.arafat alwadan my greating for u all

The patient has an epidural abscess following a dental procedure. The


epidural abscess spans from C2 to the upper thoracic spine. He has severe
neck pain, neurologic changes, and elevated laboratory markers. Sang and
associates have demonstrated that, in patients older than 65 years with a
methicillin-resistant Staphylococcus aureus infection, a history of diabetes,
and neurologic deficits, nonsurgical management has a 99% chance of
failure. Prompt surgical decompression to evacuate the abscess followed by
antibiotic treatment is the best method of treatment for this patient.
Question 53 of 100

What is the most common sequela of vertebral fracture in patients with an


ankylosed spinal disorder such as ankylosing spondylitis?
A. Noncontiguous spine fracture
B. Death
C. Wound infection
D. Spinal cord injury
Discussion: D

In a retrospective study of 112 patients with spinal fractures and an


ankylosing spinal disorder, ankylosing spondylitis or diffuse idiopathic
hyperostosis spinal cord injury was present in 58% of all patients. Of the total,
19% experienced a delay in the diagnosis of fracture. During the follow-up
period, 32% of the patients died (Caron and associates). The radiographic
findings of an ankylosed spine are readily apparent, but the specific diagnosis
of ankylosing spondylitis is often absent owing to the lack of a unified
method of diagnosis (Taurog and associates).
Question 54 of 100

CLINICAL SITUATION

Figures 1 and 2 are the MRIs from a 47-year-old man with a 1-year history of
increasing neck and arm pain on the right side more than on the left. He had

52
Done by Dr.arafat alwadan my greating for u all

a prior anterior cervical diskectomy and fusion (ACDF) performed 10 years


earlier.

When discussing the patient’s current condition, what factor do you tell him
may have contributed to its development?

A. Placement of the plate 7 mm away from the superior endplate


B. His history of smoking
C. The use of a cadaver bone rather than a polyetheretherketone cage
D. Male gender
Discussion: B

The rate of adjacent segment disease has been estimated to be between


1.6% and 4.2% per year. Placement of the plate within 5 mm of the superior
endplate has been associated with an increased risk of adjacent segment
disease. Using a cage rather than allograft has not been shown to affect
adjacent segment disease. Lee and associates showed an increased risk of
adjacent segment disease in women and smokers. They also showed a 22%
rate of revision surgery, higher than that seen in previous studies.
In a large meta-analysis, Duan and associates reported reduced short-term
and long-term rates of dysphagia in patients undergoing ACDF using a zero-
profile implant. They did not note any change in fusion rates or clinical
outcomes. Lee and associates showed equal fusion rates and segmental
motion at 1 year after surgery.
Question 55 of 83

CLINICAL SITUATION

53
Done by Dr.arafat alwadan my greating for u all

Figures 1 and 2 are the MRIs from a 47-year-old man with a 1-year history of
increasing neck and arm pain on the right side more than on the left. He had
a prior anterior cervical diskectomy and fusion (ACDF) performed 10 years
earlier.

The patient elects surgery. A zero profile anchored cage implant is selected.
Compared with conventional plate and cage (or allograft), zero profile
anchored cages have been shown to have

A. reduced rates of postoperative dysphagia.


B. reduced rates of fusion.
C. improved clinical outcomes.
D. increased segmental motion.
Discussion: A

The rate of adjacent segment disease has been estimated to be between


1.6% and 4.2% per year. Placement of the plate within 5 mm of the superior
endplate has been associated with an increased risk of adjacent segment
disease. Using a cage rather than allograft has not been shown to affect
adjacent segment disease. Lee and associates showed an increased risk of
adjacent segment disease in women and smokers. They also showed a 22%
rate of revision surgery, higher than that seen in previous studies.
In a large meta-analysis, Duan and associates reported reduced short-term
and long-term rates of dysphagia in patients undergoing ACDF using a zero-
profile implant. They did not note any change in fusion rates or clinical

54
Done by Dr.arafat alwadan my greating for u all

outcomes. Lee and associates showed equal fusion rates and segmental
motion at 1 year after surgery.
Question 56 of 100

After direct lateral (transpsoas) interbody fusion surgery at L3-4, a patient


reports numbness in the scrotum, and ipsilateral anterior thigh pain
develops. What is the most likely cause?
A. Injury to the genitofemoral nerve
B. Injury to the ilioinguinal nerve
C. Injury to the femoral nerve
D. Prolonged lateral decubitus positioning
Discussion: A

The genitofemoral nerve is at risk at almost any level in the lateral transpsoas
approach. The nerve provides sensory innervation to the anterior thigh and
scrotum/labia. The ilioinguinal nerve provides sensory innervation to the
mons pubis or labia in women and the upper scrotum in men. The femoral
nerve is responsible for sensation to the anterior and medial aspects of the
thigh, leg, and medial foot. It also provides innervation to knee extensor
muscles. Prolonged decubitus positioning, especially with jackknife
hyperextension, can cause stretching of the femoral nerve and transient
weakness of the ipsilateral quadriceps.
Question 57 of 100

A 68-year-old man with thoracic kyphosis and renal disease undergoes a


three-column thoracic osteotomy. During the osteotomy, three bilateral
segmental nerve roots are sacrificed. Before the vertebral component of the
osteotomy is begun, the patient’s motor-evoked potentials become
undetectable. What is the most likely explanation?
A. The patient has had a myocardial infarction.
B. The patient has had a cerebral vascular accident.
C. The spinal cord has sustained an ischemic event.

55
Done by Dr.arafat alwadan my greating for u all

D. The spinal cord has sustained an embolic event.


Discussion: C

Studies in dogs and subsequent follow-up studies in humans have shown


that in most patients, three levels of bilateral segmental nerve roots can be
transected safely (Fujimaki and associates, Nambu and associates, Ueda and
associates). The risk of spinal cord ischemia is higher in patients with renal
disease, however, because they are thought to be less able to compensate
for the changes in spinal cord blood flow compared with patients who have
normal renal and vascular function (Colman and associates).
Question 58 of 100

CLINICAL SITUATION

A 72-year-old-woman has leg pain on standing and walking of 1-year


duration. She has undergone physical therapy and has taken oral analgesics
with minimal relief. The patient has a history of diabetes mellitus and
coronary artery disease requiring stenting. She currently takes aspirin and
clopidogrel bisulfate (Plavix). She is offered a lumbar laminectomy.

The procedure is complicated by a small dural tear. What is the best next
step?
A. Primary repair
B. Dura seal placement
C. Bed rest
D. Fibrin glue
Discussion: A

Primary repair of incidental durotomies is the standard of care. The repair


may be supplemented by sealants such as dural seal or fibrin glue; however,
these sealants would not serve as stand-alone treatment. Bed rest may be
used postoperatively, but the duration of bed rest is controversial and not
standardized.

56
Done by Dr.arafat alwadan my greating for u all

Incidental durotomies are a common complication of lumbar spine surgery.


In the Spine Patient Outcomes Research Trial (SPORT), 409 patients
undergoing surgery for spinal stenosis had a 9% rate of incidental
durotomies. At 4-year follow-up, no difference in clinical outcome was
observed between the patients who sustained a durotomy and the group
that did not sustain a durotomy. Similar results have been noted in the SPORT
for patients undergoing lumbar microdiskectomies.
In this patient, an epidural hematoma developed, as seen on the MRI
following the resumption of her antiplatelet therapy. Clinically, she has motor
weakness and needs to return to the operating room for evacuation of the
hematoma to avoid irreversible nerve damage. She does not have an epidural
abscess, and no need exists to obtain ESR or CRP. Interventional radiology
drainage of the epidural hematoma also is not required.
Question 59 of 100

CLINICAL SITUATION

A 72-year-old-woman has leg pain on standing and walking of 1-year


duration. She has undergone physical therapy and has taken oral analgesics
with minimal relief. The patient has a history of diabetes mellitus and
coronary artery disease requiring stenting. She currently takes aspirin and
clopidogrel bisulfate (Plavix). She is offered a lumbar laminectomy.

What are the long-term outcomes of a durotomy in the setting of lumbar


stenosis and laminectomy?
A. Worse clinical outcome
B. No impact on clinical outcome
C. Higher rates of revision surgery
D. Incomplete resolution of symptoms
Discussion: B

Primary repair of incidental durotomies is the standard of care. The repair


may be supplemented by sealants such as dural seal or fibrin glue; however,
these sealants would not serve as stand-alone treatment. Bed rest may be

57
Done by Dr.arafat alwadan my greating for u all

used postoperatively, but the duration of bed rest is controversial and not
standardized.
Incidental durotomies are a common complication of lumbar spine surgery.
In the Spine Patient Outcomes Research Trial (SPORT), 409 patients
undergoing surgery for spinal stenosis had a 9% rate of incidental
durotomies. At 4-year follow-up, no difference in clinical outcome was
observed between the patients who sustained a durotomy and the group
that did not sustain a durotomy. Similar results have been noted in the SPORT
for patients undergoing lumbar microdiskectomies.
In this patient, an epidural hematoma developed, as seen on the MRI
following the resumption of her antiplatelet therapy. Clinically, she has motor
weakness and needs to return to the operating room for evacuation of the
hematoma to avoid irreversible nerve damage. She does not have an epidural
abscess, and no need exists to obtain ESR or CRP. Interventional radiology
drainage of the epidural hematoma also is not required.
Question 60 of 100

CLINICAL SITUATION

A 72-year-old-woman has leg pain on standing and walking of 1-year


duration. She has undergone physical therapy and has taken oral analgesics
with minimal relief. The patient has a history of diabetes mellitus and
coronary artery disease requiring stenting. She currently takes aspirin and
clopidogrel bisulfate (Plavix). She is offered a lumbar laminectomy.

The patient is restarted on aspirin and clopidogrel bisulfate on postoperative


day 3. In the afternoon, she is noted to have grade 4 of 5 tibialis anterior and
extensor hallucis longus, with severe back pain. An MRI is obtained and is
shown in Figure 1. On re-evaluation, she has bilateral grade 3 of 5 tibialis
anterior and extensor hallucis longus. What is the best next step?

58
Done by Dr.arafat alwadan my greating for u all

A. Obtain erythrocyte sedimentation rate (ESR), C-reactive protein (CRP) levels,


and blood cultures
B. Contact interventional radiology for drain placement
C. Return to the operating room and explore the lumbar wound
D. Observe the patient
Discussion: C

Primary repair of incidental durotomies is the standard of care. The repair


may be supplemented by sealants such as dural seal or fibrin glue; however,
these sealants would not serve as stand-alone treatment. Bed rest may be
used postoperatively, but the duration of bed rest is controversial and not
standardized.
Incidental durotomies are a common complication of lumbar spine surgery.
In the Spine Patient Outcomes Research Trial (SPORT), 409 patients
undergoing surgery for spinal stenosis had a 9% rate of incidental
durotomies. At 4-year follow-up, no difference in clinical outcome was
observed between the patients who sustained a durotomy and the group
that did not sustain a durotomy. Similar results have been noted in the SPORT
for patients undergoing lumbar microdiskectomies.
In this patient, an epidural hematoma developed, as seen on the MRI
following the resumption of her antiplatelet therapy. Clinically, she has motor
weakness and needs to return to the operating room for evacuation of the
hematoma to avoid irreversible nerve damage. She does not have an epidural
abscess, and no need exists to obtain ESR or CRP. Interventional radiology
drainage of the epidural hematoma also is not required.

59
Done by Dr.arafat alwadan my greating for u all

Question 61 of 100

What is a predictor of the clinical severity of cervical spondylotic myelopathy?


A. Age
B. Gender
C. Transverse area of the spinal cord
D. Number of stenotic levels
Discussion: C

The transverse area is the measurement of the spinal cord at the most narrow
region on the axial view of an MRI. Age, gender, and the number of stenotic
levels have not been shown to be predictors of the clinical severity of cervical
spondylotic myelopathy.
Question 62 of 100

A skeletally immature patient with adolescent idiopathic scoliosis has been


wearing a brace but only for about 6 hours per day. Her parents have tried
to coax her into wearing it 18 hours per day, and they ask you if it makes a
difference. The parents should be told that, if worn 18 hours per day, use of
the brace is
A. 10% effective.
B. 25% effective.
C. 50% effective.
D. 70% effective.
Discussion: D

A recent prospective randomized study comparing bracing to observation in


adolescent idiopathic scoliosis demonstrated that bracing was superior to
observation in preventing a curve from progressing to 50 degrees. Patients
who wore braces 18 hours per day or more were successful more than 70%
of the time (Weinstein and associates, Gomez and associates).

60
Done by Dr.arafat alwadan my greating for u all

Question 63 of 100

A 54-year-old man is diagnosed with a T6 chordoma. Which procedure can


provide the least chance of recurrence?
A. En bloc spondylectomy
B. Denosumab injection
C. Preoperative embolization
D. Postoperative radiation
Discussion: A

When feasible, en bloc resection is associated with the least recurrence in the
surgical management of chordomas. Denosumab has been used for the
treatment of giant cell tumors, along with surgical resection. Preoperative
embolization has not been associated with the prevention of recurrence.
Postoperative radiation can supplement en bloc resection but is not a stand-
alone modality that can prevent resections.
Question 64 of 100
A 30-year-old man fell off a roof. He reports neck and arm pain. On physical
examination, he has full strength in his bilateral upper extremities. CT of his
cervical spine reveals a C6 ipsilateral lamina and pedicle fracture. What is the
recommended treatment?
A. One-level anterior cervical diskectomy and fusion (ACDF)
B. Two-level ACDF
C. Hard collar
D. Halo vest application
Discussion: B

The patient has sustained a floating lateral mass injury. These injuries have a
high risk of subsequent displacement. Surgical stabilization is recommended.
Surgery includes a two-level anterior cervical diskectomy with instrumented
fusion. Halo vest application is a poor immobilizer of subaxial cervical injuries.

61
Done by Dr.arafat alwadan my greating for u all

Question 65 of 100
Figure 1 is the radiograph from a 20-year-old mountain biker who lost
control on a trail. When he was brought to the emergency department by
paramedics, he was alert and oriented. He reported neck pain and painful
tingling in bilateral hands. He has no motor weakness and has normal
sensation. What is the best next step?

A. Application of traction with Gardner Wells tongs


B. Heavy sedation for relaxation and application of a halo vest with traction
C. Emergent cervical MRI
D. Immediate initiation of spinal cord injury steroid protocol
Discussion: A

This patient has bilateral C5-6 facet perch. He is awake, oriented, and able to
be examined. The patient is also an appropriate candidate for reduction by
serial traction. Because of his normal mental status, he is able to convey the
symptoms of neurologic deterioration due to iatrogenic disk herniation
during the reduction maneuver. Sedating the patient may compromise his
ability to accurately perform serial exams and will affect the patient's ability
to communicate symptomatic neurologic deterioration. Cervical MRI is not
always readily available and can delay reduction. MRI can be performed after
facet reduction. The patient does not have a spinal cord injury; thus, the
steroid protocol is not indicated.

62
Done by Dr.arafat alwadan my greating for u all

Question 66 of 100
Which type of thoracolumbar injury typically involves all three columns, is
most mechanically unstable, and is most associated with complete spinal
cord injury?
A. Compression
B. Burst
C. Flexion distraction
D. Translation-rotation
Discussions: D

Translation-rotation injuries typically yield fracture dislocations. This injury


pattern involves the disruption of skeletal and ligamentous elements of the
spine to cause a maximum loss of stability, subsequent deformity in three
planes (coronal, axial, and sagittal), and catastrophic neurologic injury.
Compression injuries occur when a force is applied in flexion and injures the
anterior column. Compression injuries are usually stable and rarely have
neurologic sequelae. Burst fractures occur through axially applied forces,
which in turn cause injury to the anterior and middle columns of the
vertebrae at minimum. Neurologic injury can occur through direct
compression of the neural elements by bone fragments or hematoma or by
absorption of the transferred energy. Flexion distraction injuries typically
occur as forces are transmitted from anterior to posterior, causing injury to
the middle and posterior columns.
Question 67 of 100
CLINICAL SITUATION

Figures 1 through 3 show MRI and CT images from a 56-year-old man with
known metastatic lung cancer who came to the emergency department with
increasing back pain. On examination, he has grade 4+ of 5 strength in the
bilateral lower extremities. He has intact rectal tone and volitional
contraction.

How is the stability of his spine determined?

63
Done by Dr.arafat alwadan my greating for u all

A. Thoracolumbar Injury Classification System (TLICS)


B. Denis Classification System
C. Kostuik Classification System
D. Spinal Instability Neoplastic Score (SINS)
Discussion: D

Determining spinal instability is critical in the setting of metastatic or primary


spine tumors. The SINS is a multidisciplinary validated score that can guide
treatment. SINS has six components, and a score of 6 or less describes a
stable spine, 7 to 12 describes impending instability, and 13 and above
defines an unstable spine. The TLICS is used for surgical decision making in
traumatic thoracolumbar spine fractures. The Denis classification also is used
for decision making in spine fractures. The Kostuik classification is an axial–
CT-based classification used to determine the extent of tumor involvement
in a vertebral body.
Following spine tumor surgery, wound complications such as infection or
dehiscence are the most common complications. Prior radiation treatment
and revision surgery can increase the risk of wound complications. Dural
tears, instrumentation failure, and tumor recurrence are also complications
of spine tumor surgery but are less frequent than wound complications.
Stereotactic radiosurgery is a radiation therapy modality that can provide
focused treatment with higher doses of radiation than conventional
radiotherapy. Compared with conventional radiotherapy, less fractionation
of treatment is needed. One of the complications of stereotactic radiosurgery

64
Done by Dr.arafat alwadan my greating for u all

is an 11% to 39% rate of compression fractures. Osteoradionecrosis is the


proposed mechanism for the compression fractures.
The patient has some radicular symptoms, and CT demonstrates pedicle and
vertebral body involvement. Transpedicular decompression provides
adequate access and decompression of the pedicle and vertebral body.
Lumbar laminectomy alone would not provide access to the vertebral body.
L4 corpectomy is not indicated to address this metastatic spine disease.
Lateral extracavitary decompression is an option when attempting all
posterior based corpectomies in the lumbar spine but is not indicated for this
patient.
Question 68 of 100
CLINICAL SITUATION

Figures 1 through 3 show MRI and CT images from a 56-year-old man with
known metastatic lung cancer who came to the emergency department with
increasing back pain. On examination, he has grade 4+ of 5 strength in the
bilateral lower extremities. He has intact rectal tone and volitional
contraction.

The patient’s SINS is 9, or impending instability. He is offered surgical


stabilization. What is the most common surgical complication that should be
anticipated?

A. Dural tear
B. Instrumentation failure
C. Tumor recurrence

65
Done by Dr.arafat alwadan my greating for u all

D. Wound complications

Discussion: D
Determining spinal instability is critical in the setting of metastatic or primary
spine tumors. The SINS is a multidisciplinary validated score that can guide
treatment. SINS has six components, and a score of 6 or less describes a
stable spine, 7 to 12 describes impending instability, and 13 and above
defines an unstable spine. The TLICS is used for surgical decision making in
traumatic thoracolumbar spine fractures. The Denis classification also is used
for decision making in spine fractures. The Kostuik classification is an axial–
CT-based classification used to determine the extent of tumor involvement
in a vertebral body.
Following spine tumor surgery, wound complications such as infection or
dehiscence are the most common complications. Prior radiation treatment
and revision surgery can increase the risk of wound complications. Dural
tears, instrumentation failure, and tumor recurrence are also complications
of spine tumor surgery but are less frequent than wound complications.
Stereotactic radiosurgery is a radiation therapy modality that can provide
focused treatment with higher doses of radiation than conventional
radiotherapy. Compared with conventional radiotherapy, less fractionation
of treatment is needed. One of the complications of stereotactic radiosurgery
is an 11% to 39% rate of compression fractures. Osteoradionecrosis is the
proposed mechanism for the compression fractures.
The patient has some radicular symptoms, and CT demonstrates pedicle and
vertebral body involvement. Transpedicular decompression provides
adequate access and decompression of the pedicle and vertebral body.
Lumbar laminectomy alone would not provide access to the vertebral body.
L4 corpectomy is not indicated to address this metastatic spine disease.
Lateral extracavitary decompression is an option when attempting all
posterior based corpectomies in the lumbar spine but is not indicated for this
patient.
Question 69 of 100
CLINICAL SITUATION

66
Done by Dr.arafat alwadan my greating for u all

Figures 1 through 3 show MRI and CT images from a 56-year-old man with
known metastatic lung cancer who came to the emergency department with
increasing back pain. On examination, he has grade 4+ of 5 strength in the
bilateral lower extremities. He has intact rectal tone and volitional
contraction.

The patient is offered surgical intervention with the understanding that he


will undergo radiation treatment 3 weeks postoperatively. Instead of
conventional radiation therapy, however, stereotactic radiosurgery is
recommended. What is a common complication of stereotactic radiosurgery?

A. Nerve damage
B. Compression fracture
C. Tumor recurrence
D. Increased pain
Discussions: B

Determining spinal instability is critical in the setting of metastatic or primary


spine tumors. The SINS is a multidisciplinary validated score that can guide
treatment. SINS has six components, and a score of 6 or less describes a
stable spine, 7 to 12 describes impending instability, and 13 and above
defines an unstable spine. The TLICS is used for surgical decision making in
traumatic thoracolumbar spine fractures. The Denis classification also is used
for decision making in spine fractures. The Kostuik classification is an axial–
CT-based classification used to determine the extent of tumor involvement
in a vertebral body.

67
Done by Dr.arafat alwadan my greating for u all

Following spine tumor surgery, wound complications such as infection or


dehiscence are the most common complications. Prior radiation treatment
and revision surgery can increase the risk of wound complications. Dural
tears, instrumentation failure, and tumor recurrence are also complications
of spine tumor surgery but are less frequent than wound complications.
Stereotactic radiosurgery is a radiation therapy modality that can provide
focused treatment with higher doses of radiation than conventional
radiotherapy. Compared with conventional radiotherapy, less fractionation
of treatment is needed. One of the complications of stereotactic radiosurgery
is an 11% to 39% rate of compression fractures. Osteoradionecrosis is the
proposed mechanism for the compression fractures.
The patient has some radicular symptoms, and CT demonstrates pedicle and
vertebral body involvement. Transpedicular decompression provides
adequate access and decompression of the pedicle and vertebral body.
Lumbar laminectomy alone would not provide access to the vertebral body.
L4 corpectomy is not indicated to address this metastatic spine disease.
Lateral extracavitary decompression is an option when attempting all
posterior based corpectomies in the lumbar spine but is not indicated for this
patient
Question 70 of 100
CLINICAL SITUATION

Figures 1 through 3 show MRI and CT images from a 56-year-old man with
known metastatic lung cancer who came to the emergency department with
increasing back pain. On examination, he has grade 4+ of 5 strength in the
bilateral lower extremities. He has intact rectal tone and volitional
contraction.

The patient has been given a life expectancy of 6 months. What


decompression technique can achieve sufficient decompression in the
setting of his metastatic disease?

68
Done by Dr.arafat alwadan my greating for u all

A. L4 corpectomy
B. Lumbar laminectomy
C. Transpedicular decompression
D. Lateral extracavitary decompression
Discussion: C

Determining spinal instability is critical in the setting of metastatic or primary


spine tumors. The SINS is a multidisciplinary validated score that can guide
treatment. SINS has six components, and a score of 6 or less describes a
stable spine, 7 to 12 describes impending instability, and 13 and above
defines an unstable spine. The TLICS is used for surgical decision making in
traumatic thoracolumbar spine fractures. The Denis classification also is used
for decision making in spine fractures. The Kostuik classification is an axial–
CT-based classification used to determine the extent of tumor involvement
in a vertebral body.
Following spine tumor surgery, wound complications such as infection or
dehiscence are the most common complications. Prior radiation treatment
and revision surgery can increase the risk of wound complications. Dural
tears, instrumentation failure, and tumor recurrence are also complications
of spine tumor surgery but are less frequent than wound complications.
Stereotactic radiosurgery is a radiation therapy modality that can provide
focused treatment with higher doses of radiation than conventional
radiotherapy. Compared with conventional radiotherapy, less fractionation
of treatment is needed. One of the complications of stereotactic radiosurgery

69
Done by Dr.arafat alwadan my greating for u all

is an 11% to 39% rate of compression fractures. Osteoradionecrosis is the


proposed mechanism for the compression fractures.
The patient has some radicular symptoms, and CT demonstrates pedicle and
vertebral body involvement. Transpedicular decompression provides
adequate access and decompression of the pedicle and vertebral body.
Lumbar laminectomy alone would not provide access to the vertebral body.
L4 corpectomy is not indicated to address this metastatic spine disease.
Lateral extracavitary decompression is an option when attempting all
posterior based corpectomies in the lumbar spine but is not indicated for this
patient.
Question 71 of 100
Figure 1 shows an axial T2-weighted MRI from a 25-year-old man with
severe right leg pain from the buttock to the anterolateral aspect of the lower
leg, with associated numbness. The pain has been present for 3 months in
spite of physical therapy and nonsteroidal anti-inflammatory medication.
What is the most compelling reason to consider a lumbar epidural steroid
injection (ESI) in this patient?

A. ESIs have been shown to provide durable pain relief when compared with
usual care.
B. ESIs have been shown to reduce acute radicular pain.
C. ESIs can reduce the likelihood of surgical decompression.
D. ESIs are less expensive than surgery.

70
Done by Dr.arafat alwadan my greating for u all

Discussion: B

The use of epidural steroids has not been shown to change the natural
history of lumbar radiculopathy secondary to a herniated nucleus pulposus.
ESI has been shown to provide improvements in pain over the short term,
but these gains are not discernable over time when compared with patients
treated without ESI (Chou and associates). Most patients improve without
surgery; in fact, nearly 90% of patients improve by 3 months (Deyo and
associates).
Question 72 of 100
Figures 1 and 2 are the MRI and CT images from a 73-year-old woman with
mild neck pain and balance difficulty. She reports worsening ambulation and
hand dexterity over the past 2 years. The patient currently uses a walker to
ambulate. On examination, she has a positive Hoffman sign and a positive
Romberg sign. She has no motor deficits. Surgery is offered to the patient.
What is a common complication encountered during anterior surgical
management of this condition?

A. Paralysis
B. Recurrence of stenosis
C. Dural tear
D. Instrumentation failure
Discussion: C

The patient has ossification of the posterior longitudinal ligament (OPLL).


Cervical dural tears are uncommon, with a prevalence of 1%. In OPLL,
however, the rate can be as high as 12.5%. Paralysis, instrumentation failure,

71
Done by Dr.arafat alwadan my greating for u all

and recurrent stenosis all are risks following cervical spine surgery, but
increased rates of durotomies are unique to OPLL.
Question 73 of 100
Figures 1 and 2 are the MRIs obtained from a 58-year-old woman who has
symptoms of neurogenic claudication. You elect to treat the patient with a
lateral lumbar interbody fusion with posterior pedicle screw instrumentation
but no direct neural decompression. When deciding on this treatment
option, you consider that

A. a 20% chance exists that the indirect decompression alone will not be
sufficient and that a decompression procedure may be needed in the future.
B. indirect decompression results in a 10% to 20% increase in the spinal canal
area.
C. placement of the cage closer to the midpoint of the disk (in the sagittal plane)
will result in a larger increase in the spinal canal area.
D. indirect decompression without direct decompression has been shown to
produce better postoperative functional outcome scores.
Discussion: B

In degenerative spondylolisthesis, indirect decompression of the spinal canal


has been shown to be an effective treatment option. Malham and associates
conducted a prospective study of 122 patients and reported an unplanned
return to the operating room in 11 patients (9%). When reviewing these cases
retrospectively, the authors felt that failure of indirect decompression should
have been anticipated based on radiographic findings in 10 of these 11
patients who had high-grade, unstable spondylolisthesis or substantial bony

72
Done by Dr.arafat alwadan my greating for u all

lateral recess stenosis. Sato and associates reported an increase in the spinal
canal area of 20%, whereas Castellvi and associates reported only a 9%
increase. Park and associates reported that positioning the cage within the
anterior one-third of disk space is better for achieving the restoration of the
segmental angle without compromising the indirect neural decompression,
if the cage was high enough.
Question 74 of 100
An 83-year-old woman has leg pain with ambulation. She has tried physical
therapy, oral analgesics, and injections, with minimal relief. The symptoms
have been present for 1 year. Radiographs reveal an L4-5 spondylolisthesis
and greater than 4 mm of motion on flexion-extension. MRI shows moderate
to severe central and lateral recess stenosis. The patient should be informed
that at her age, surgical intervention
A. can provide considerable benefit compared with nonsurgical management.
B. should be avoided because of a higher rate of complications.
C. has no difference in results from nonsurgical management.
D. can lead to increased mortality.
Discussion: A

The incidence of surgery is increased in patients 80 years of age and older.


Patients aged 80 years and older enrolled in the Spine Patient Outcomes
Research Trial and undergoing surgery for lumbar stenosis and
spondylolisthesis were compared with patients younger than 80. In the older
age group, surgical treatment was associated with statistically significant
clinical improvement compared with nonsurgical management. No
statistically significant increase was observed in complications or mortality
compared with younger patients.
Question 75 of 100
Figure 1 is a sagittal T2-weighted MRI obtained from a 78-year-old man in
the emergency department following a fall in which he struck the front of his
head and extended his neck. On physical examination, he is noted to have

73
Done by Dr.arafat alwadan my greating for u all

reduced sensation in his bilateral upper and lower extremities, grade 2 of 5


strength in his bilateral upper extremities, and grade 5 of 5 strength in his
bilateral lower extremities. When counseling the patient and his family on
treatment options, you tell them that

A. advanced age does not affect neurologic outcome.


B. delayed surgery is associated with longer intensive care unit (ICU) and
hospital stays.
C. delayed surgery has been associated with reduced mortality rates.
D. delayed surgery has been associated with improved neurologic recovery.
Discussion: C

The timing of surgery for central cord syndrome remains controversial.


Although the study by Anderson and associates suggested that early surgery
may lead to better neurologic recovery, other studies have shown that the
timing of surgery does not have any effect. Samuel and associates
demonstrated reduced mortality with delayed surgery. They also reported
that delayed surgery was associated with an increase in minor adverse
events, but no change in serious adverse events was observed. Kepler and
associates showed that advanced age is associated with poorer neurologic
outcomes. They also observed no change in length of stay or ICU days
between early and late surgical groups.
Question 76 of 100
For the clinical situation described, match the most appropriate
treatment listed.

74
Done by Dr.arafat alwadan my greating for u all

Figure 1 shows a T2-weighted MRI from a 72-year-old man with known


metastatic prostate cancer. The arrows demonstrate metastatic epidural
prostate disease. The patient comes to the emergency department with
difficulty walking and leg numbness. He is found to have lower extremity
weakness on examination.

A. Palliative radiation
B. En bloc spondylectomy
C. Separation surgery followed by stereotactic radiation
D. Observation
E. Stereotactic radiosurgery
F. Kyphoplasty
Discussion: C

Radiation remains the standard of care for most symptomatic spinal


metastasis. In the absence of cord compression or instability, radiation is the
best choice. Palliative dose radiation is unlikely to reverse the neurologic
decline that has started for the first patient, however. This patient needs more
aggressive treatment. Insufficient clearance exists between the tumor and
the spinal cord to allow the safe delivery of stereotactic radiosurgery. Surgery
should be performed to create space between the tumor and the cord. This
so-called “separation” surgery enables the safe use of stereotactic
radiosurgery postoperatively (Laufer and associates). The situation is
different for primary tumors of the spine. Although radiation has improved
and many clinicians use radiation to treat chordomas in combination with
surgery, the standard of care is en bloc spondylectomy (Boriani and
associates).

75
Done by Dr.arafat alwadan my greating for u all

Question 77 of 100
For the clinical situation described, match the most appropriate
treatment listed.

Figure 2 depicts an axial T2-weighted MRI from a 58-year-old man with 6


months of progressive pain in his back and legs. The image reveals a large
mass that was biopsied and found to be consistent with chordoma. The pain
keeps him from sleeping. He has no other disease sites.

A. Palliative radiation
B. En bloc spondylectomy
C. Separation surgery followed by stereotactic radiation
D. Observation
E. Stereotactic radiosurgery
F. Kyphoplasty
Discussion: B

Radiation remains the standard of care for most symptomatic spinal


metastasis. In the absence of cord compression or instability, radiation is the
best choice. Palliative dose radiation is unlikely to reverse the neurologic
decline that has started for the first patient, however. This patient needs more
aggressive treatment. Insufficient clearance exists between the tumor and
the spinal cord to allow the safe delivery of stereotactic radiosurgery. Surgery
should be performed to create space between the tumor and the cord. This
so-called “separation” surgery enables the safe use of stereotactic
radiosurgery postoperatively (Laufer and associates). The situation is
different for primary tumors of the spine. Although radiation has improved

76
Done by Dr.arafat alwadan my greating for u all

and many clinicians use radiation to treat chordomas in combination with


surgery, the standard of care is en bloc spondylectomy (Boriani and
associates).
Question 78 of 100
For the clinical situation described, match the most appropriate
treatment listed.

Figure 3 shows an MRI from a 72-year-old woman with known breast


carcinoma and liver metastasis who has pain at the base of her neck. A biopsy
was performed that confirmed the diagnosis of metastatic breast cancer in
the first thoracic vertebrae, which was not present on an MRI from 1 year
earlier. What is the best course of action?

A. Palliative radiation
B. En bloc spondylectomy
C. Separation surgery followed by stereotactic radiation
D. Observation
E. Stereotactic radiosurgery
F. Kyphoplasty
Discussion: A

Radiation remains the standard of care for most symptomatic spinal


metastasis. In the absence of cord compression or instability, radiation is the
best choice. Palliative dose radiation is unlikely to reverse the neurologic
decline that has started for the first patient, however. This patient needs more
aggressive treatment. Insufficient clearance exists between the tumor and

77
Done by Dr.arafat alwadan my greating for u all

the spinal cord to allow the safe delivery of stereotactic radiosurgery. Surgery
should be performed to create space between the tumor and the cord. This
so-called “separation” surgery enables the safe use of stereotactic
radiosurgery postoperatively (Laufer and associates). The situation is
different for primary tumors of the spine. Although radiation has improved
and many clinicians use radiation to treat chordomas in combination with
surgery, the standard of care is en bloc spondylectomy (Boriani and
associates).
Question 79 of 100
During the listed surgery, which of the structures in the responses is at
greatest risk of injury?

Removal of bone during cervical corpectomy


A. Ansa cervicalis
B. Vagus nerve
C. Recurrent laryngeal nerve
D. Greater occipital nerve
E. C5 nerve root
F. Vertebral artery
G. Internal carotid artery
Discussion: F

Increased risk to the vertebral artery occurs during cervical corpectomy due
to an aberrant course of the vertebral artery. The incidence of midline
migration of the vertebral artery is 7.6%. The vertebral artery is also at
increased risk during posterior C1 arch exposure, which should be limited to
1.5 cm lateral to the midline. During anterior midcervical spine exposure, the
recurrent laryngeal nerve is at risk and nerve injury<strong> </strong>can
lead to postoperative dysphagia. The internal carotid artery lies just anterior
to the anterior arch of C1 and is at risk during bicortical fixation with either a
C1 lateral mass or C1-2 transarticular fixation. During posterior laminectomy
and instrumented fusion, multiple structures are at low risk, but the greatest

78
Done by Dr.arafat alwadan my greating for u all

risk is of C5 nerve palsy, which occurs in approximately 7% of cases and has


reported rates ranging from 0% to 30%.
Question 80 of 100
During the listed surgery, which of the structures in the responses is at
greatest risk of injury?

Anterior midcervical spine exposure


A. Ansa cervicalis
B. Vagus nerve
C. Recurrent laryngeal nerve
D. Greater occipital nerve
E. C5 nerve root
F. Vertebral artery
G. Internal carotid artery
Discussion: C

Increased risk to the vertebral artery occurs during cervical corpectomy due
to an aberrant course of the vertebral artery. The incidence of midline
migration of the vertebral artery is 7.6%. The vertebral artery is also at
increased risk during posterior C1 arch exposure, which should be limited to
1.5 cm lateral to the midline. During anterior midcervical spine exposure, the
recurrent laryngeal nerve is at risk and nerve injury<strong> </strong>can
lead to postoperative dysphagia. The internal carotid artery lies just anterior
to the anterior arch of C1 and is at risk during bicortical fixation with either a
C1 lateral mass or C1-2 transarticular fixation. During posterior laminectomy
and instrumented fusion, multiple structures are at low risk, but the greatest
risk is of C5 nerve palsy, which occurs in approximately 7% of cases and has
reported rates ranging from 0% to 30%.
Question 81 of 100
During the listed surgery, which of the structures in the responses is at
greatest risk of injury?

Bicortical placement of C1-2 transarticular screw

79
Done by Dr.arafat alwadan my greating for u all

A. Ansa cervicalis
B. Vagus nerve
C. Recurrent laryngeal nerve
D. Greater occipital nerve
E. C5 nerve root
F. Vertebral artery
G. Internal carotid artery
Discussion: G

Increased risk to the vertebral artery occurs during cervical corpectomy due
to an aberrant course of the vertebral artery. The incidence of midline
migration of the vertebral artery is 7.6%. The vertebral artery is also at
increased risk during posterior C1 arch exposure, which should be limited to
1.5 cm lateral to the midline. During anterior midcervical spine exposure, the
recurrent laryngeal nerve is at risk and nerve injury<strong> </strong>can
lead to postoperative dysphagia. The internal carotid artery lies just anterior
to the anterior arch of C1 and is at risk during bicortical fixation with either a
C1 lateral mass or C1-2 transarticular fixation. During posterior laminectomy
and instrumented fusion, multiple structures are at low risk, but the greatest
risk is of C5 nerve palsy, which occurs in approximately 7% of cases and has
reported rates ranging from 0% to 30%.
Question 82 of 100
During the listed surgery, which of the structures in the responses is at
greatest risk of injury?

Lateral exposure of C1 posterior arch


A. Ansa cervicalis
B. Vagus nerve
C. Recurrent laryngeal nerve
D. Greater occipital nerve
E. C5 nerve root
F. Vertebral artery
G. Internal carotid artery

80
Done by Dr.arafat alwadan my greating for u all

Discussion: F

Increased risk to the vertebral artery occurs during cervical corpectomy due
to an aberrant course of the vertebral artery. The incidence of midline
migration of the vertebral artery is 7.6%. The vertebral artery is also at
increased risk during posterior C1 arch exposure, which should be limited to
1.5 cm lateral to the midline. During anterior midcervical spine exposure, the
recurrent laryngeal nerve is at risk and nerve injury<strong> </strong>can
lead to postoperative dysphagia. The internal carotid artery lies just anterior
to the anterior arch of C1 and is at risk during bicortical fixation with either a
C1 lateral mass or C1-2 transarticular fixation. During posterior laminectomy
and instrumented fusion, multiple structures are at low risk, but the greatest
risk is of C5 nerve palsy, which occurs in approximately 7% of cases and has
reported rates ranging from 0% to 30%.
Question 83 of 100
During the listed surgery, which of the structures in the responses is at
greatest risk of injury?

Cervical laminectomy and fusion


A. Ansa cervicalis
B. Vagus nerve
C. Recurrent laryngeal nerve
D. Greater occipital nerve
E. C5 nerve root
F. Vertebral artery
G. Internal carotid artery
Discussion: E

Increased risk to the vertebral artery occurs during cervical corpectomy due
to an aberrant course of the vertebral artery. The incidence of midline
migration of the vertebral artery is 7.6%. The vertebral artery is also at
increased risk during posterior C1 arch exposure, which should be limited to
1.5 cm lateral to the midline. During anterior midcervical spine exposure, the

81
Done by Dr.arafat alwadan my greating for u all

recurrent laryngeal nerve is at risk and nerve injury<strong> </strong>can


lead to postoperative dysphagia. The internal carotid artery lies just anterior
to the anterior arch of C1 and is at risk during bicortical fixation with either a
C1 lateral mass or C1-2 transarticular fixation. During posterior laminectomy
and instrumented fusion, multiple structures are at low risk, but the greatest
risk is of C5 nerve palsy, which occurs in approximately 7% of cases and has
reported rates ranging from 0% to 30%.
Question 84 of 100
For the following statement, match the most appropriate tumor type
listed.

Most likely to respond to radiation


A. Multiple myeloma
B. Non-small-cell lung carcinoma
C. Prostate cancer
D. Breast cancer
E. Squamous cell carcinoma
F. Renal cell carcinoma
Discussion: A

The management of metastatic bone disease in the spine requires an


understanding of the biology of common metastatic tumors in bone.
Multiple myeloma is actually a blood-born tumor that is associated with
substantial lytic bone lesions. It also can be associated with soft-tissue
masses that compress the spinal cord. It is among the most radiation-
sensitive tumors (Biermann). Prostate cancer is the best example of a purely
osteoblastic tumor (Edeiken and associates). Non–small-cell lung carcinoma
carries a very poor prognosis, and survival has been reported to be just over
3 months after spinal decompression (Goodwin and associates). Renal cell
carcinoma should be considered for embolization prior to surgery to help
mitigate bleeding, for which it is rightly associated (Quraishi and associates)

82
Done by Dr.arafat alwadan my greating for u all

Question 85 of 100
For the following statement, match the most appropriate tumor type
listed.

Most likely to present as densely blastic on radiograph


A. Multiple myeloma
B. Non-small-cell lung carcinoma
C. Prostate cancer
D. Breast cancer
E. Squamous cell carcinoma
F. Renal cell carcinoma
Discussion: C

The management of metastatic bone disease in the spine requires an


understanding of the biology of common metastatic tumors in bone.
Multiple myeloma is actually a blood-born tumor that is associated with
substantial lytic bone lesions. It also can be associated with soft-tissue
masses that compress the spinal cord. It is among the most radiation-
sensitive tumors (Biermann). Prostate cancer is the best example of a purely
osteoblastic tumor (Edeiken and associates). Non–small-cell lung carcinoma
carries a very poor prognosis, and survival has been reported to be just over
3 months after spinal decompression (Goodwin and associates). Renal cell
carcinoma should be considered for embolization prior to surgery to help
mitigate bleeding, for which it is rightly associated (Quraishi and associates).
Question 86 of 100
For the following statement, match the most appropriate tumor type
listed.

Most likely to lead to patient’s death in 6 months or less


A. Multiple myeloma
B. Non-small-cell lung carcinoma
C. Prostate cancer
D. Breast cancer

83
Done by Dr.arafat alwadan my greating for u all

E. Squamous cell carcinoma


F. Renal cell carcinoma
Discussion: B

The management of metastatic bone disease in the spine requires an


understanding of the biology of common metastatic tumors in bone.
Multiple myeloma is actually a blood-born tumor that is associated with
substantial lytic bone lesions. It also can be associated with soft-tissue
masses that compress the spinal cord. It is among the most radiation-
sensitive tumors (Biermann). Prostate cancer is the best example of a purely
osteoblastic tumor (Edeiken and associates). Non–small-cell lung carcinoma
carries a very poor prognosis, and survival has been reported to be just over
3 months after spinal decompression (Goodwin and associates). Renal cell
carcinoma should be considered for embolization prior to surgery to help
mitigate bleeding, for which it is rightly associated (Quraishi and associates).
Question 87 of 100
For the following statement, match the most appropriate tumor type
listed.

Most likely to benefit from preoperative embolization


A. Multiple myeloma
B. Non-small-cell lung carcinoma
C. Prostate cancer
D. Breast cancer
E. Squamous cell carcinoma
F. Renal cell carcinoma
Discussion: F

The management of metastatic bone disease in the spine requires an


understanding of the biology of common metastatic tumors in bone.
Multiple myeloma is actually a blood-born tumor that is associated with
substantial lytic bone lesions. It also can be associated with soft-tissue
masses that compress the spinal cord. It is among the most radiation-

84
Done by Dr.arafat alwadan my greating for u all

sensitive tumors (Biermann). Prostate cancer is the best example of a purely


osteoblastic tumor (Edeiken and associates). Non–small-cell lung carcinoma
carries a very poor prognosis, and survival has been reported to be just over
3 months after spinal decompression (Goodwin and associates). Renal cell
carcinoma should be considered for embolization prior to surgery to help
mitigate bleeding, for which it is rightly associated (Quraishi and associates).
Question 88 of 100
Figures 1 and 2 are MRI images obtained from a 22-year-old man who fell
from a 2-story building. On examination, he has diminished rectal tone and
urinary retention. If surgical stabilization is elected, what is the most
biomechanically stable option?

A. Sacral plating
B. Iliosacral screws
C. Iliosacral screws and lumbopelvic fixation
D. External fixation
Discussion: C

The patient has a U-shaped sacral fracture or spondylopelvic dissociation.


Treatment options for these fractures range from percutaneous placement
of iliosacral screws to lumbopelvic fixation (lumbar pedicle screws and iliac
screws). Lumbopelvic fixation can be supplemented by iliosacral screws,
which has been termed triangular osteosynthesis. Biomechanical studies
have shown that iliosacral screws with lumbopelvic fixation—or triangular

85
Done by Dr.arafat alwadan my greating for u all

osteosynthesis—is the most stable construct when compared with iliosacral


screws alone. The advantage of lumbopelvic fixation is that concurrent sacral
laminectomy can be performed, which is recommend in this patient because
of his neurologic symptoms. External fixation or sacral plating play minimal
roles in U-shaped sacral fractures.
Question 89 of 100
Figures 1 and 2 show the MRIs from a 20-year-old woman who was involved
in a motor vehicle collision and has C4 and C5 fractures. On examination, she
displays diminished rectal tone and sensation, with grade 3 of 5 biceps and
deltoid strength and 0 of 5 motor strength in the triceps and below. What
management option would most likely increase her motor function?

A. High-dose steroids
B. Maintaining mean arterial pressure above 85 mm Hg
C. Delayed cervical decompression and stabilization
D. Cervical decompression and stabilization within 24 hours
Discussion: D

Traumatic cervical spinal cord injury is managed best with surgical


decompression and stabilization performed within 24 hours to maximize the
possibility of neurologic recovery. The Surgical Timing in Acute Spinal Cord
Injury Study noted improvement in Asia Impairment Scale scores in patients
undergoing decompression within 24 hours of injury when compared with
those managed in a delayed manner (Fehling and associates). The use of
high-dose steroids in the setting of spinal cord injury has declined
considerably nationally because of corticosteroid-related complications.
Although elevating mean arterial pressures can help increase spinal cord

86
Done by Dr.arafat alwadan my greating for u all

perfusion, the practice is an adjunct to prompt decompression and


stabilization rather than an isolated treatment option.
Question 90 of 100
CLINICAL SITUATION

Figures 1 through 3 show the MRI and CT images from a man who sustained
a blunt force trauma after a motorcycle accident 10 hours earlier.

What factors contribute the most to the decision to operate?

A. The percentage of spinal canal compromise and the degree of kyphosis


B. The fracture morphology on radiographic imaging, neurologic status, and the
integrity of the diskoligamentous complex
C. The fracture classification on CT, the presence of hematoma on MRI, and the
level of consciousness
D. The neurologic status and coexisting injuries
Discussion: B

The patient has an unstable burst fracture. According to the Thoracolumbar


Injury Classification and Severity (TLICS) classification, the fracture
morphology, neurologic function of the patient, and integrity of the
diskoligamentous complex all contribute to a scoring system designed to
facilitate the decision to operate to decompress and stabilize the fracture.
Concomitant injuries ultimately may play a role in the timing of surgery, but
they usually do not affect the decision to operate. The level of consciousness

87
Done by Dr.arafat alwadan my greating for u all

of the patient likewise may affect the timing of surgery, but not the decision
to operate.
This patient has an incomplete spinal cord injury with compression of the
cord. The fracture morphology, level of neurologic impairment, and disk
injury with anterior compression give him a TLICS score higher than 5 (8 to 9
by calculation), meeting the criteria for surgical treatment. As seen on
radiographic imaging, the greatest compression of the neural elements
comes from the anterior fragments of bone and disk, which can be
decompressed most predictably through an anterior approach. A
translational and sagittal deformity is present, which needs to be addressed
from a circumferential approach.
No role exists for bracing alone in a neurologically impaired patient with an
unstable fracture pattern. Posterior distraction instrumentation using
ligamentotaxis alone has been found to indirectly decompress the canal,
depending on the integrity of the remaining ligamentous structures. The
amount of this decompression is unpredictable and inferior to direct
decompression. Short-segment stabilization from a posterior alone
approach also has been reported to have complications with late-onset
deformity.
Question 91 of 100
CLINICAL SITUATION

Figures 1 through 3 show the MRI and CT images from a man who sustained
a blunt force trauma after a motorcycle accident 10 hours earlier.

A patient who was admitted to the intensive care unit reports severe back
pain. On physical examination, he displays grade 2 of 5 weakness in bilateral
hip flexion, bilateral quadriceps, and trace strength (1 of 5) in ankle
dorsiflexion and plantar flexion. His sensory examination is remarkable for
decreased sensation below the bilateral inguinal regions down to the toes.
He has reduced rectal sensation and tone. The bulbocavernosus reflex is
present. What is the best next step?

88
Done by Dr.arafat alwadan my greating for u all

A. Thoracolumbosacral orthosis bracing and intravenoussteroids


B. Posterior percutaneous thoracolumbar distractioninstrumentation
C. Posterior transpedicular decompression with instrumented fusion fromT 12-
L2
D. Anterior L1 corpectomy with placement ofastrutgraft and posterior
instrumentedfusion from T12-L2 Commented [aa1]:

Discussion: D

The patient has an unstable burst fracture. According to the Thoracolumbar


Injury Classification and Severity (TLICS) classification, the fracture
morphology, neurologic function of the patient, and integrity of the
diskoligamentous complex all contribute to a scoring system designed to
facilitate the decision to operate to decompress and stabilize the fracture.
Concomitant injuries ultimately may play a role in the timing of surgery, but
they usually do not affect the decision to operate. The level of consciousness
of the patient likewise may affect the timing of surgery, but not the decision
to operate.
This patient has an incomplete spinal cord injury with compression of the
cord. The fracture morphology, level of neurologic impairment, and disk
injury with anterior compression give him a TLICS score higher than 5 (8 to 9
by calculation), meeting the criteria for surgical treatment. As seen on
radiographic imaging, the greatest compression of the neural elements
comes from the anterior fragments of bone and disk, which can be
decompressed most predictably through an anterior approach. A

89
Done by Dr.arafat alwadan my greating for u all

translational and sagittal deformity is present, which needs to be addressed


from a circumferential approach.
No role exists for bracing alone in a neurologically impaired patient with an
unstable fracture pattern. Posterior distraction instrumentation using
ligamentotaxis alone has been found to indirectly decompress the canal,
depending on the integrity of the remaining ligamentous structures. The
amount of this decompression is unpredictable and inferior to direct
decompression. Short-segment stabilization from a posterior alone
approach also has been reported to have complications with late-onset
deformity.
Question 92 of 100
A 54-year-old man sustains an H-type sacral fracture in a motor vehicle
collision. Resuscitation has been completed. He reports back pain. A
thorough neurologic examination is performed. What predicts neurologic
dysfunction?
A. Age
B. Gender
C. Displacement of the fracture
D. Mechanism of injury
Discussion: C

The displacement of sacral fractures confers an increased risk of neurologic


dysfunction. Age, gender, and mechanism of injury have not been shown to
affect the neurologic status substantially.
Question 93 of 100
Figure 1 is the radiograph from an 18-year-old man who arrives in the
emergency department after a high-speed motorcycle collision. Fluid
resuscitation was started immediately upon ambulance transport. His
hemodynamic status includes blood pressure at 100/70 and pulse at 80 beats
per minute. The respiratory rate is 20 breaths per minute and shallow. The
urine output is 30 ml per hour, and intravenous fluids are flowing at 250 ml
per hour. On physical examination, he is found to be insensate below the

90
Done by Dr.arafat alwadan my greating for u all

level of injury. The motor strength status is revealed to be flaccid paralysis


below the level of injury. His bulbocavernosus reflex is negative. The mental
status examination shows that he is alert and anxious. What best describes
his condition?

A. Neurogenic shock
B. Spinal shock
C. Hypovolemic shock
D. Septic shock
Discussion: B

The displacement of sacral fractures confers an increased risk of neurologic


dysfunction. Age, gender, and mechanism of injury have not been shown to
affect the neurologic status substantially.
Question 94 of 100
Figures 1 through 3 represent the MRI images from a 28-year-old man who
was ejected from a car and sustained a cervical spine injury. He has no motor
or sensory function below C5. In comparison to spinal cord injury without
facet dislocation, jumped facets are associated with

91
Done by Dr.arafat alwadan my greating for u all

A. a worse clinical outcome.


B. no impact on the clinical outcome.
C. higher rates of revision surgery.
D. an incomplete resolution of symptoms.
Discussion: A

In this sense, shock does not refer to circulatory shock. In spinal shock, a
spinal cord injury results in a loss of motor and sensory function below the
level of injury, along with a total loss of reflexes. Spinal shock usually starts
minutes after injury and can last 24 to 72 hours.
Question 95 of 100
Figures 1 through 4 are from a 20-year-old woman brought to the
emergency department after a motorcycle collision. She has a blood alcohol
level of 0.2. Fluid resuscitation was started by emergency medical personnel,
and she has 2 large bore intravenous infusions with, crystalloid running at
250 ml per hour. She has received 2 liters of crystalloid since the accident.
Her blood pressure is 78/40, and her pulse is 122 beats per minute and is
weak and thready. Her skin is pale, cool, and moist. A Foley catheter is placed,
and her urine output is 20 ml per hour. Her mental status is altered and
ranges from somnolent to anxious. Her motor examination ability is
decreased in the lower extremities, and sensation is decreased below the
umbilicus. The initial complete blood count drawn in the trauma bay returns
with a white blood cell count of 11.5 and hemoglobin (Hgb) levels of 14.5%.
What best describes her condition?

92
Done by Dr.arafat alwadan my greating for u all

A. Neurogenic shock
B. Spinal shock
C. Hypovolemic shock
D. Septic shock

Discussion: C

This patient sustained a multitraumatic injury and is suffering from


hypovolemic shock. Despite infusion of isotonic intravenous fluids, she
remains hypotensive, tachycardic (appropriately), and oliguric. Her
abdominal CT shows a splenic injury with hematoma. She has sustained T12
and L3 vertebral injuries, but these injuries typically would not cause
hypotension. Hypovolemic shock is characterized by low blood pressure,
tachycardia, low urine output, hemoconcentration (high to spuriously normal
Hgb or hematocrit), mental status changes, and skin changes (cool, clammy,
and pale). She requires full resuscitation, more crystalloid, or volume
expansion (albumin or blood products).
Question 96 of 100
CLINICAL SITUATION

93
Done by Dr.arafat alwadan my greating for u all

Figures 1 and 2 show the imaging studies from a 75-year-old man brought
to the emergency department with upper neck pain after a ground-level fall.
The patient is a resident of a skilled nursing facility who ambulates minimally
with a walker. He has severe chronic obstructive pulmonary disorder and is
on continuous oxygen. He is alert and oriented and has normal sensation in
the upper and lower extremities. His examination is positive for weakness
and pain on right shoulder movement but is otherwise normal.

What is the best next diagnostic study?

A. Flexion-extension radiographs of the cervical spine


B. CT of the brain
C. Radiographs of the right upper extremity
D. Electromyogram/nerve conduction study of the upper extremities
Discussion: C

This patient has a comminuted fracture of the arch of the C1 ring. He has a
lateral mass displacement of less than 7 mm, making transverse ligament
injury less likely. A Jefferson burst fracture is unlikely to cause isolated
shoulder weakness and pain. Imaging of the upper extremity can help
identify other injuries that may explain the patient’s weakness such as a
clavicle or proximal humerus fracture. For this fracture type, application of a
hard collar or halo vest is appropriate, but this patient is a poor candidate for
a halo vest because of his physiologic status. Surgical intervention is not
required for either fracture type, because the C1 fracture can be treated
closed and the distal clavicle injury is minimally displaced. A soft collar is less

94
Done by Dr.arafat alwadan my greating for u all

likely to provide adequate stability for a comminuted Jefferson burst fracture


of C1.
Question 97 of 100
CLINICAL SITUATION

Figures 1 and 2 show the imaging studies from a 75-year-old man brought
to the emergency department with upper neck pain after a ground-level fall.
The patient is a resident of a skilled nursing facility who ambulates minimally
with a walker. He has severe chronic obstructive pulmonary disorder and is
on continuous oxygen. He is alert and oriented and has normal sensation in
the upper and lower extremities. His examination is positive for weakness
and pain on right shoulder movement but is otherwise normal.

What is the most appropriate management of the patient’s cervical spine?

A. Soft collar application


B. Hard collar application
C. Halo vest application
D. Posterior C1-2 fusion
Discussion: B

This patient has a comminuted fracture of the arch of the C1 ring. He has a
lateral mass displacement of less than 7 mm, making transverse ligament
injury less likely. A Jefferson burst fracture is unlikely to cause isolated
shoulder weakness and pain. Imaging of the upper extremity can help
identify other injuries that may explain the patient’s weakness such as a
clavicle or proximal humerus fracture. For this fracture type, application of a
hard collar or halo vest is appropriate, but this patient is a poor candidate for
a halo vest because of his physiologic status. Surgical intervention is not

95
Done by Dr.arafat alwadan my greating for u all

required for either fracture type, because the C1 fracture can be treated
closed and the distal clavicle injury is minimally displaced. A soft collar is less
likely to provide adequate stability for a comminuted Jefferson burst fracture
of C1.
Question 98 of 100
Figures 1 and 2 show the radiograph and MRI from an 18-year-old man who
is brought into the emergency department after falling off his skateboard
and hitting his head against a wall. He was wearing a helmet. He denies loss
of consciousness but reports severe neck pain. His motor and sensory
function are intact. What is the best next step?

A. Placement in a Minerva brace


B. Anterior odontoid screw fixation
C. Gentle reduction using halo vest traction and posterior C1-3 arthrodesis
D. Posterior wiring of C2-3 along with placement of bone graft
Discussion: C

This patient has a type IIA Hangman fracture or traumatic spondylolisthesis


of the axis. The preferred treatment of this displaced fracture is surgical:
reduction (traction versus open), followed by posterior instrumentation and
fusion. Using a hard collar is acceptable for a type I minimally displaced
injury. The Minerva brace is an orthosis reserved for lower cervical or
cervicothoracic junction injuries. Anterior odontoid screw fixation is reserved
for type II odontoid injuries, which are amenable to an anterior approach.

96
Done by Dr.arafat alwadan my greating for u all

Posterior spinous process or laminar wiring will not stabilize this injury,
because the fracture is at the pars interarticularis.
Question 99 of 100
A 56-year-old man is brought to the emergency department by paramedics
following a high-speed motor vehicle collision. He has obvious head trauma
as seen by bilateral periorbital ecchymoses, substantial facial swelling, and a
large bitemporal scalp laceration. He is not alert, but he is responsive to
painful stimuli, and he moves all four extremities. What radiographic test
would you order first to assess his cervical spine for potential injury?
A. A cross-table lateral cervical spine plain radiograph
B. A complete cervical spine series
C. CT of the cervical spine
D. Cervical MRI
Discussion: C

CT of the cervical spine is fast and readily available in most centers. The
reported sensitivity of CT is greater than 95%, whereas specificity is almost
100%. In contrast, plain radiographs have a sensitivity of 70% and a missed
injury rate of 15% to 30%. CT also has been found to be as cost effective or
more cost effective compared with plain radiographs in diagnosing cervical
injuries. MRI is expensive, not always readily available, and inferior to CT in
diagnosing bony injuries. In this patient, dynamic imaging in the form of
flexion-extension views is contraindicated as a first line radiographic test. The
patient may have an unstable cervical injury which could be exacerbated with
motion. The patient's mental status also does not allow voluntary motion.
The maneuver would have to be done by the physician or radiology
technician.
Question 100 of 100
Figures 1 and 2 show CT images from a 24-year-old man who was the
unrestrained driver in a single motor vehicle collision. By report, he was
ejected from the vehicle and initially was found unresponsive. The patient

97
Done by Dr.arafat alwadan my greating for u all

was intubated in the field and then brought by ambulance to the emergency
department, where he was resuscitated aggressively with crystalloid and
blood transfusions. Radiographs were taken and showed an intracranial
hemorrhage, which required emergent burr hole evacuation by
Neurosurgery. In the intensive care unit, his blood pressure is 80/48, and his
pulse is 48. He is breathing spontaneously on the ventilator at 16 breaths per
minute. He can follow commands. Physical examination reveals absent motor
function in the legs, no sensation below the nipple level, and a positive
bulbocavernosus reflex. His skin is warm and dry. What best describes his
condition?

A. Neurogenic shock
B. Spinal shock
C. Hypovolemic shock
D. Septic shock
Discussion: A

This patient has classic neurogenic shock, which usually occurs when a
cervical or high thoracic cord injury disrupts the autonomic pathways and
causes a loss of sympathetic tone. Characteristic hypotension and
bradycardia are present due to an unopposed vagal tone. Low cardiac output
also is present, along with venous and arterial dilatation. The treatment for
neurogenic shock is administration of agents called pressors (phenylephrine,
dopamine, dobutamine, and norepinephrine) to improve cardiac contractility
and increase peripheral vascular resistance. Atropine is given to increase the
heart rate. Pressors are titrated to keep the mean arterial pressure above 80
and maintain spinal cord perfusion.

98
Done by Dr.arafat alwadan my greating for u all

99

Potrebbero piacerti anche